SlideShare une entreprise Scribd logo
1  sur  81
Télécharger pour lire hors ligne
CrnprpR


                                                                          Applications
                                                                          of Integrals



                                                          OYERVIEW Many things we want to know can be calculated with integrals: the
                                                          areasbetweencurves,the volumes and sudaceareasof solids, the lengthsof curves,
                                                         the amount of work it takes to pump liquids fiom belowground, the forces againsr
                                                         floodgates,the coordinatesof the points where solid objects will balance.We define
                                                         all of these as limits of Riemann sums of continuous functions on closed intervals.
                                                         that is, as integrals,and evaluatethese limits with calculus.
                                                              There is a pattern to how we define the integrals in applications, a pattern
                                                         that, once learned,enablesus to define new integralswhen we need them. We look
                                                         at specific applications 0rst, then examine the pattern and show how it leads to
                                                         integrals in new situations.




                                                        AreasBetweenCurves
                                                        This section shows how to lind the areas of regions in the coordinate plane bv
                                                        integrating the functions that define the regions, boundaries.


                                                        The Basic
                                                                Formula a Limitof Riemann
                                                                      as                Sums
                                                        Supposewe want to find the area of a region that is bounded above by the curve
                                                        -y =.l(r), below by the curve ) = g(.r), and on the left and right by the lines
                                                        .r : 4 and r : D (Fig. 5.1). The region might accidentallyhave a shapewhose area
                                                        w e c o u l d f i n L w i r h g e o m e t r l b u r i l / a n d g a r e i t r b i l r a r ) o n r i n u o ut,u n c r r o n,r
                                                                              l                       .                                           c                                  e
                                                        usuaily have to flnd the area with an integral.
                                                             To see what the integml should be, we lirst approximate the region with,?
                                                                                    basedon a parlition p : {.r0, ... , r,,} of [ri, D] (Fig. 5.2, on
                                                        verticalrectangles                                                      rr,
                                                        the following page). The area of the tth rectangle(Fig. 5.3, on rhe followrng page;
                                                        ls

                                                                                  AA1 : tsjcfi1 x width : Ll (c.t) g(ci)l A4.
                                                        We then approximatethe area of the region by adding the areasof the ,?rectanqles:

                                                                                      A* f          ^A, -Itr,,.r                    grr.,rlA,:,.
'    The region between y: f(x) and
                                                        As ll P | -+ 0 the sums on rhe right approachthe limit
y:   g ( x ) a n d t h e l i n e sx : a a n d x = b .                                                                                                 _ g(,r)l d,r because
                                                                                                               /j t/(r)

                                                                                                                                                                             365
Chapter5: Applicationsof Integrals
356




                                                                                            T      - s(ck)
                                                                                            f(t)




                                                                                                   Stctll

::.: We approximate regionwith
                  the
          perpendicular the x-axis.
                      to                             AAk = areaof kth rectangle,f(c*) - g(cr) = height, lxk = width
rectangles



                                                                                                       this integral'
                                           /and g arecontinuous. takethearea theregionto be thevalueof
                                                                            of
                                                              We
                                            That is,

                                                             lim )-l/(c,)-gtcrtlAxr - fo rf ,r,-
                                                       A-                                                    Stxt]dx
                                                            nP o--                          J




                                              Definition
                                                                     with /(r) > g(x) throughout b], ihenthe area
                                                      g                                        [4,
                                              Ifl ancl arecontinuous
                                                                   the curves : /(*) andy : g(t) ftom a to b is the
                                                                             y
                                              of the regionbetween
                                              integral [/ - 8 | liom a to D:
                                                       o[
                                                                          rb
                                                                                 U(x)- g(x)ldx.                  (1)
                                                                       A=
                                                                            J


                                               To apply Eq. (1) we take the following steps.



                                               How to Find the Area Between Two Curves
                                                  Graph the cunes and draw a tepresentative rectangle. Thts rcveals
                                                l.
                                                  which curve is / (upper curve) and which is g (1owercurye). It also
                                                  helps find the limits of integration if you do not aheady know them.

      ,s'I ,=sinr                             2. Find the limits of integration.




          I--''-
                                               3. Write a formula for f(x) - g(x). Simplify it if you can.
                                               4. lntegrate [f(x) - gQ, fron a to b Tl1e number you get is the area.




         H                                                     Find the area between lv = sec2 and y : 3i1;5 from 0 to z/4'
                                                                                              x
                                            EXAMPLE 1

                                            Solution
                                            Step t; We sketch the cutves and a vertical rectangle(Fig. 5.4). The upper curve
                                            is the gmph of /(;r) = secz;r: the lower is the graph of g(;) : sipI'
                                            Sfep 2; The limits of integration are already given: a :0' b = r /4'
 ,'.]].The regionin Example'lwith a                                        -
                                             SteP3: f (x) - 8(x) = sec2;r sin't
                       rectangle.
 typicalapproximating
5.1 AreasBetweenCurves 367

                                  Step 4:

                                                               {r' *- sinx)r/x : ftan + cos
                                                                                      -r    x]i/a
                                                          lo'o

                                                          f'.+l,to+:+
                                                                -t
                                                          L                                                                                  f,

                                  Curves
                                       That Intersect
                                  When a region is detemined by curves that intersect, the intersection points give
                                  the limits of integration.


                                                                                                                                         - x2
                                                    Find the area of the region enclosedby the parabola ! :2
                                  EXAMPLE 2
                                                 : -y.
                                  and the line y

                                  Solution
                                  Sfep t; Sketch the curves and a vertical rectangle (Fig. 5.5). Identifying the upper
                  (-{,,f(-r)
                                                                                              -x. The ,{-coordinates
                                  and the lower curyes, we take /(x) :2-              g(r):
                                                                              xz arLd
                                                    pointsare the limits of integration.
                                  of the intersecdon
                                  Step 2; We find the limits of integation by solving | :2 - xz and y : -1 ri-
( 1 ,1 )
                                  multaneously for .r.'
                                                                                             -x       [ q r l L t/r1 r ) . u r d . q ( f r
                                                                            2-xz :

                                                                      x2-x-2:0                        R.rir..
                                                                 (x+l)(x -2):0                        Fxrr.'
               (a s(r))
                                                                      -1,
                                                                 x:                 x :2.             sot.

                                  The region runs from x : - 1 to.r : 2. The limits of integrationare a : - l, b : 2.
                                  Step 3:

5.5 The regionin Example with a
                        2
typicalapproximatingrectangle.


                                  Step 4
                                                                t2                     f        -2                           -t12
                                                      -     :
                                       o:        rf,.,>e(x)ldx I t z + x - x z t d x = l z r +  z
                                                                                                 -                           ,I
                                             l'                                                                             JJ
                                                                      J-t                              I                             r
                                                                      /         4       8        /        |      1
                                                                      (o*r-r,l (-'*l*r/
                                                                            ?qq
                                                                            -
                                                                      -'2
                                                                      At

                                                                                    3        2                                               l

                                      Technofogy The Intersectionof Two Graphs One of the difficult and some-
                                      times frustrating parts of integmtion applications is finding the limits of inte-
                                      gration. To do this you often have to find the zeroes of a function or the
                                      intenection points of two curues.
                                                                         g(r) using a graphing utility, you enter
                                         To solve the equation /(x):
                                                               yr: f@)          and y2: g@)
Chapter Applications lntegrals
                         of
            5:
368

                                                                                                                     you
                                            andusethe grapherroutine to find the Pointsof intersection'Altematively'
                                                                       - g(T) : 0 with a root finder'Try bothprocedures
                                            lun solve*tJ         /(:r)
                                                         quation
                                            with
                                                                              and g(x):3-x'
                                                               f (x):lnx
                                                                                                                   hidden
                                             When points of intersectionarc not clearly revealedor you suspect
                                                                                                   further use of calculus
                                             behavior,additional work with the graphing utility or
                                             may be necessary,




                                                    ISECT
                                                                    =.79205996845


                                                                 curves : Inx and lz:3 - x' usinga built-infundion
                                                                       y1
                                             a) The intersecting
                                                            intersection
                                                to find the                                            -
                                                                                                   :        x
                                             O) Usinga built-inroot finder to find the zero of f(x) Inx 3 +



                                                with ChangingFormulas
                                       Boundaries
                                                                                                 points' we partition
                                       If the formula for a bounding curve changesat one or mole
                                       th,gionintosubregionsthatconespondtotheformulachangesandapplyEq.
                                       (1) to eachsubregion.


                                                        Find the areaof the region in the fint quadrantthat is bounded
                                       EXAMPLE 3
                                                                                                   - 2'
                                                                                                :
                                       aboveby y : .r/i and below by the t-axis and the line y x

                                       solution
                                                                                                          is the graphof
                                       Step t.' The sketch(Fig. 5.6) showsthat the region'supperboundary
                                                                                                          <2to g(x) =
                                                                                  fromg(;r):0for0 S r
                                                                         changes
                                       f(; : JV. Theloweiboundary
                                         -' Z tot Z : t : 4 (thereis agreement r : 2)' We partition the region at r : 2
                                                                               at
                                                                                            rectanglefor eachsubregion'
                                       
                                       into subregiJns ani f and tketh a representative
                                                        A




 5.6 When the formulafor a boundingcurve
 changes, area integralchanges match
                               to
        the
 (Example3).
                                                    'fe limits of integrationfor regionA area : O andb :,2' The lefchand
                                            step 2:
                                           li#t for region B rs a = 2. To find the right-hand limit, we solve
                                                                                                              the equations
5.1 AreasBetween
                                                                                                                           Curves 369


                                        y : rG andy : x - 2 simultaneously r.'
                                                                          for
                                                                                                                         EqratcI (.r)rnd
                                                                                             ^-
                                                                                         -        z
                                                                                   v^
                                                                                                                         .9(11
                                                                                        a:(x.-2)2:x.2-4x+4                SqLrafeboth

                                                                  x 2- 5 x + 4 : o                                        Re$'r'ite.

                                                             (.r-1)(r-4):0                                               FacL(r'.

                                                                                   -_1                -_i
                                                                                                                         Solve.

                                        Only the value.x=4 satisfies equation
                                                                          the                             The valuer:1is                    an
                                                                                            Ji:x-2.
                                        extraneous introduced squaring. right-hand
                                                        root          by           The                limit is b:4.
                                                                       f (x)- S@):             -0:
                                        S t e p3 . ' F o r 05 x = 2 :
                                                                            - e ( x ) : , 6E - ( x - 2q :
                                                                                                    )
                                                                                                            Ji - x +2
                                                    F o r2 a x a 4 :   f(x)
                                        Step 4.' We add the areaof subregions and B to find the total area:
                                                                            A
                                                               f)-f4
                                                            =
                                              T o t a fr e a | J i d x I l t J x - x + 2 t d x
                                                     a
                                                              -o -  - -  - /
                                                             J                      Jz
                                                                                              areaof B
                                                               arca of A

                                                                          'l2
                                                             f)                    f1             -2          14

                                                             fil.+l1*'''-v+2,1,
                                                                                       /2 -     
                                                             ?ef,,_o_ (1,,0,',, .' 8)_
                                                                            _8
                                                             r         r          , r{zt'''-z++1
                                                             ?,t, : !.
                                                                -,                                 ,
                                                             33

                                        lntegrationwith Respect y
                                                              to
                                        If a region's bounding curves are describedby functions of y, the approximating
                                        rectanglesare horizontal insteadof vertical and the basic formula has y in place
                                        of r.




                                                                                    t


                                                                                    1
                                                                                    I
                                                                                    L*,,,

                                           use tJteformula
In Eq. (2),/always denotes right-hand
                            the
                                                                                                  - stytldy.
                                                                         e:
                                                                                   l.' vrrt
                                                                                                                                       (2)
curve and I the left-hand curve, so
 fO) - e(y) is nonnegative.
Chapter 5: Applicationsof lntegrals
370


                                                                Find the area of the region in Example 3 by integrating with
                                              EXAMPLE 4
                                              rcspect to ).

                                              Solution
      (c()),
           ).)
                              x:y+2           Step t; We sketchthe region and a typical horiz,ontalrectanglebasedon a panition
                                              of an interval ofy values(Fig. 5.7). The region's right-handboundary is the line x :
                                              y f 2, so l (y) : y + 2. The left-hand boundary is the curve ir : y2, so g(y) : y2.
                                              Step 2: The lower limit of integration is l = 0. We find the upper lirnit by solving
                                              I : y *2 and r: )2 simultaneously y;   for

5.7 lt takestwo integrations find the                                                    l+t:l
                             to
areaof this regionif we integrate  with
                                                                                             2=o
                                                                                  ,t-)
respectto x It takes only one if we
          with respect y (Example
integrate             to             4).                                      ( ) + 1 ) ( ) 2 ): 0           Iurt,,

                                                                                                 1'-
                                                                                   _t                  )     S ,i l  .

                                              The upperlimit of integmtion D : 2. (The valuey : -1 givesa point of inter-
                                                                         is
                                              sectlor
                                                     belowthe r-axis.)
                                              Step3:
                                                                                =)+2            y2=2iy            )?
                                                                  /())-s())
                                              Step 4:

                                                                      v,rt-    :
                                                               e:
                                                                    1,,'' srt,rat 1 2 + y - y ' ) l d y
                                                                                 l,'
                                                                                                  f        r,2       rt l'
                                                                                            :      +;               1l
                                                                                               L2Y                   3 ln
                                                                                                             8            l0
                                                                                            - o.+ t -
                                                                                                4
                                                                                                                          3
                                                                                                             3
                                              This is the result of Example 3, fbund with less work.


                                                      Integrals
                                                              with Formulas
                                                                          from Geometry
                                              Combining
                                                                                                   andgeometry.
                                              The fastest
                                                        way to find an areamay be to combinecalculus

                                                                The Area of the Region in Example 3 Found the Fastesi
                                              EXAMPLE5
                                              way
                                              Find the area of the region in Example 3.

                                              So/ution The area we want is the areabetweenthe curye I : Ji,O = x S 4, and
             1:rf                             the x-axis, minus the area of a triangle with base 2 and height 2 (Fig. 5.8):
                                  2                                                                    lr2rr2r
                        /                                                x r u = f or t a ,
                             .2
                         '.2
                                                                                   2 ,11
                                                                                   J   .lo
5.8 The areaof the blue regionis the
                                                                               :3ttl-o -'z:+'
area underthe parabola = 1& minus
                                                                                                                                 ,
                        y
the areaof the triangle.                                                           JJ




                                                                                                                           -
Exercises
                                                                                                             5.1    371


                                                Moral of Examples3-5 It is sometimes   easierto lind the areabetween two
                                                                  with respect y instead ,r. Also, it may help to combine
                                                curves integrating
                                                      by                      to        of
                                                geometry calculus.
                                                         and        After sketching region,takea moment determine
                                                                                  the                     to          the
                                                bestwav to oroceed.




Exercises
        5.1
Find the areasof the shadedregions in Exercises 1-8.
 l.                                2.
Chapter Applications Integrals
                            of
               5:
372

                                                                   16. y : x 2 - 2 x       and Y=r
                              area.
In Exercises findthetotalshaded
          9-12,
                                                                                       and y = -tt2 14x
                                                                   t7. ) : 12
                          10.
 9,
            ]                                                      18. y = l - l x '            ano ]:x-14
  (-3,5)                                                           19. y = x 4 - 4 x 2 + 4            and Y=a2
                  I                                                                                           and y:0
                                                                   20, y = r n @ = 7 ,              a>0,
         _,_,2
                                                                                                   5y = x * 6 (How many intersection points are
                                                                   21. y = {fl             nd
                                                                         there?)
                                              2x3 -                                                         (x212)+4
                                                           5x      22,y:lx2-41'and                     ):
                                                      2
                      I
                                                                   Find the areasof the regions enclosedby the lines and curves
                                                                            23-30.
                                                                   Exercises
                           -3)                                                   : r : 0 , a n d) : 3
                                                                       x:2y2,
                                                                   23.
                                                                   24. x = y2 u1t4 a: y 12
                                                                                  and' 4x - Y = 16
                                                                   25. y2 - 4x:4
                                                                   26, x - y2 :0 arld x +2Y2 =3
                                                                                            ^nd r+3Y2:2
                                                                   2 7 ,x * y 2 : O
                                                                                                 and ,r*)a:2
                                                                    2 8 .x - y 2 / t : 0
(-2,4
                                                                                                                     y
                                                                   29.x =y2-l           and x='yJl
                                                                   3 0 . r = ) l 3- y 2 a n d x = 2 y

      -2 -l                                                        Find the areasof the regions enclosedby the curves in Exercises

      I                                                            31'34.
                                                                                                 and .xa-Y:l
                                                                    31.4x2+y:4
   I                                                                                            a n d 3 x 2- y : 4
                                                                                 y :O
                                                                    3 2 .x 3
   I,                       (3,5)                                                                                        for xZ0
                                                                                                 and :r*)a:1,
                                                                    3 3 .x * 4 y 2 : 4
                                                                                                and 4.t*y2:0
                                                                    3 4 ,x a y z : 3
12.
                                                                    Find the arcas of the regions enclosedby the lines and curves in
              )

              I+.
                                                                             35-42.
                                                                    Exercises
                          (3,6)
          6
                                                                                            and y = sin2.r, 0:.x57t
                                                                    3 5 .y = 2 s 1 n x
                                                                                            and y:5e92.r, -7t13=x Sr13
          v                                                         36.):8cos.r
                                                                                                                     .t2
                                                                                                    and 1:1
                                                                    3 7 .y : c o s ( r x / 2 )
                                                                    38. y = sin(nx/2) and 1t:r
                          (3,1)                                                                      and'x:tt/4
                                                                    39, l:sec2.r, y=tarfx, x:-tr/4,
                                                                                  and .x:-tan2), -7r14=r 3n/4
                                                                    4O. :ta#y
                                                                        x


 r                                                                                                  and -x=0, 0=y<7t/2
                                                                    4 1 .r : 3 s i n l / i o s y
   -'
                                                                     4 2 .y : s s s 2 1 1 t a J 3 1 r r d y : r 1 / 1 , - l = i r : l
          3)                                                                                      a
                                                                                                           rcgion enclosed the curve
                                                                                                                         by
                                                                    43. Find the areaof the propeller-shaped
 Find the areasof the regions enclosedby the lines and curves in          - y3 : 0 andthe line.r - Y : 0.
                                                                        t
            13-22.
 Exercises                                                                                                       region enclosedby the
                                                                    44. Find the area of the propeller-shaped
 1 3 . y : a 2- 2                                                       clruesx -yll3 : 0 andr - )l/5 = 0.
                   ar'd y:2
 14. y =2x - x2 arLd Y: -J                                          45, Find the areaof the region in the first quadrantboundedby the
                                                                        line I :1, the line,t :2, the curve1 : 1/-r'?, the -t-aris.
                                                                                                                         and
 15. y :1+ and ) :8r
Exercises
                                                                                                                                                        5.1   373


                                                                                 Suppose area of the region betweenthe graph of a positive
46. Find the area of the tdangular region in the first quadrant                        the
                                                                                 continuousfunction / and the .x-axisfrom x : 4 to x : b is 4
    boundedon the left by the y-axis and on the right by the curves
                                                                                        units. Find the areabetween curves)r: /(.r) and
                                                                                                                      the
    ) = sin.rand): cos,r.                                                        square
                                                                                 y-2f(x)ftomx:atox-b.
                          belowby the pambola : ,r2 andabove       by
47, The regionbounded                            )
                                                                                Which of the following integals, if either calculates areaof
                                                                                                                                     the
               : 4 is to be parlitioned into two subsections equal
                                                             of
    the line )
                                                                                          region shownhere?Give reasonsfor your answer.
    areaby cutting acrossit with the horizontal line ), = c.                    the shaded
    a) Sketchthe region and draw a line ) : c acrossit that looks                         fL                               f)
                                                                                              -l-x)d:(=
                                                                                         I (x                                       2xdx
                                                                                 al                                        |
          about right. In terms of c, what are the coordinates the
                                                                of                                                     J
                                                                                        J-t                                     I

          points where the line and parabolaintersect?Add them to                                    '
                                                                                          rot                              tl
                                                                                                                                    -zxax
                                                                                 bt      / r-x-(r))dx-
          your figure.                                                                                                     |
                                                                                                                       JI
                                                                                        J-t
    b) Find c by integratingwith respectto ),. (This puts c in the
          limits of integration.)
                                             to.x.(Thisputsc into the
    c) Find c by integntingwith respect
          integrand well.)
                     as
                                                               ''2 and
    Find the areaof the regionbetween curve )):3
                                            the
    the line y = 1 by integratingwith respectto (a) t, (b) ).
49. Find the areaof the region in the first quadrantboundedon the
    left by the ),-axis,below by the line y = t /4, aboveleft by the
    curve y = I * .,[, and aboveright by he (i.jtr..te : 2/ Ji,
                                                        y
50. Find the area of the region in the first quadrantboundedon the
    left by the y-axis, below by the curve r : 2/t, aboveleft by
                                         right by the line i :3 - ).
    the curyer : () - l)'?,andabove
                                                                           54. True, sometimestue, or never tue? The area of the region
                                                                               between graphsof the continuous     functions): /(i) and
                                                                                       the
                                                                                 : g(r) andthe verticallinesr : a andx: b (a < b) rs
                                                                               )
                                                                                                 rb
                                                                                                I lf @) - s(x)ld)c.
                                                                                                           J

                                                                                 Give reasonsfor your answer.




                                                                         S CASExplorationsand Proiects
                                                                           In Exercises 55-58, you will find the area between curves in the plane
                                                                           when you cannot find their points of intersectionusing simple algebra.
                                                                           Use a CAS to perform the following steps:
    The figure here shows triangle AOC inscribed in the region cut
                                                                            a) Plot the curves together to see what they look like and how many
    ftom the parabola ) - .r2 by the line y: a2. Find the limit of
                                                                                                      (hey hae.
                                                                               points of inrersection
    the latio of the area of the triangle to the area of the parabolic
                                                                            b) Use the numerical equation solver in your CAS to find all the
    region as 4 approaches zerc.
                                                                               points of intersection.
                                                                                                - g(r)l over consecutive pairs of intersection
                                                                            c) Integate
                                                                                          f(r)
                                                                               values.
                                                                            d) Sum together the integrals found in part (c).

                                                                                                ^-       ^ -2,     I           . g 1 x .:1 - J
                                                                                                                                          x
                                                                           5 5 . '/3 2 3 :
                                                                                   1x1

                                                                                                t4
                                                                                                                1 0 .g ( . r r : 8 - l 2 r
                                                                           5 6 ./ t x t : i - t r ' -

                                                                           57. /(r) =x *sin(2tc), g(x) : 13
                                                                           58. /(.x) : -r2sqsir, g(tc) : x1 - x
Chapter 5: Applicationsof Integrals
374




                                                           Volumes Slicing
                                                                 by
                                                     Finding
                                                     From the areasof regions with curved boundaries,we can calculatethe volumes of
                                                     cylinders with curved basesby multiplying base area by height. From the volumes
                                                     of such cylinders, we can calculate the volumes of other solids.


                                                     5licing
                                                     Supposewe want to find the volume of a solid like the one shown in Fig. 5.9. At
            Cross sectionR(jr). Its area is A(ir).
                                                     each point r in the closed inteNal la, rl the cross section of the solid is a region
                                                     R(x) whose area is A(r). This makesA a real-valuedfunction of .r' If it is also a
                                                     continuous lunction of .r, we can use it to define and calculate the volume of the
                                                     solid as an integral in the following way.
                                                          We partition the interval [4, ]l along the r-a{is in the usual manner and slice
                                                     the solid, as we would a loaf of bread, by planes petpendicular to the -r-axis at
                                                     the partition points. The tth slice, the one between the planes at xk r and tn, has
                                                     approximatelythe same volume as the cylinder betweenthesetwo planes basedon
                                                     the region R(,rr) (Fig. 5.10). The volume of this cylinder is

                                                                Vr : base area x height
                                sedion
     lf the areaA(x) of the cross
                                                                    : A(rr) x (distancebetween the planes at -rr r and xr)
                     functionof x, we can
R(x)is a continuous
find the volumeof the solid bY                                      :   A(xr)Axr.
             A(x) from a to b.
integrating
                                                     The volume of the solid is therelbre approximatedby the cylinder volume sum
                                                                                              -    r,. r,r-.
                                                                                              ?-,'^'^
                                                     This is a Riemann sum for the function A(r) on [c, Dl. We expectthe approxlmahons
                                                     from these sums to improve as the norm of the partition of la, bl goes to zero, so
                                                     we define their limiting integral to be the volume of the solid.



                                                                                                    Approxrmatlng
                                                                            Plane at.r[   I         cllinderbased   p l a n en r r ,
                                                                                                    on]((rt]




                                                                                                           -'i.---------'
                                                                                                                      -,
                                                                               The cylinder's base
                                                                               is the region R(,v*).
                                                                                                   NOTTO SCAI-E


                                                                  view of the sliceof the solidbetweenthe
                                                     .   Enlarged
                                                                                               cylinder'
                                                     planes xk r and xk and its approximating
                                                           at
5 . 2 F i n d i n g o l u m eb y S l i c i n g 3 7 5
                                                                                                                     V          s



                                         Definition
                                        The volume of a solid of known integrable cross-sectionarea A(x) from
                                        x = a to x: D is the integralofA from a to r.

                                                                                   v : fo e61a,.                                             (j)
                                                                                            J



                                         T o a p p l l E q . { l ) . w e r a k et h e f o l l o w i n g: r e p . .



                                         How to FindVolumesby the Method of Slicing
                                        1     Sketch the solid and a typical cross section.
                                        2.    Find a formula for A(r).
                                        3.    Find the limits of integration.
                                        4.    Integrate A(x) to find the volume.




                                     EXAMPLE 7         A pyramid 3 m high has a square base that is 3 mon a side.
                                     The cross section of the pyramid perpendicularto the altitude r m down from the
                                     ve ex is a squarer m on a side. Find the volume of the pyramid.

                                     Solution
                                     Step 1: A sketcll. We draw the pyramid with its altitude along the r-axis and its
                                     vertexat the origin and includea typical crosssection(Fig. 5.11).

                                                                          Typical cfoss
                                                                               section



                                                                   --r$€t
                                                                                                     -=_,,.,



             sections the pyramidin Example
     The cross      of                    1
are 5quare5.

                                     Step 2: A.fonnula for A(x). The cross section at r is a squarex meters on a side,
                                     so lts area ls

                                                                                       A(r) : 12'

                                     Step 3: Tlle lintits of integration. The squaresgo from r :0                           to;r :3
                                     Step 4: The volwne.
                                                                                                               'l'
                                                                        lb                        t' -      r.
                                                                                                        -,
                                                                             At^trlt -
                                                              v=I                                | t:dr     r |=9
                                                                      J..                     J,,                    .l ,
                                     The volume is 9 mr.
Chapter Applications Integrals
                             of
376             5:

                                                                           wedge cut from a cylinderof radius3 by two planes.
                                                                                   is
                                                            2     A curved
                                                EXAMPLE
Bonaventura Cavalieri ('l 598-1647)                                                                                   planecrosses
                                                One planeis perpendicular the axis of the cylinder.The second
                                                                            to
Cavalied, a studentof Galileo's, discovered     the nrst plane at a 45' angle at the centerof the cylinder. Find the volume of the
that if two plane rcgions can be afiangedto     wedge.
lie over the sameinterval of the i-axis 1n
sucha way that they haveidentical vertical      Solution
crosssectionsat every point, then the rcgions                                                                   perpen-
                                                Step 1: A sketch. draw the wedgeand sketcha typical crosssection
                                                                 We
havethe sarnearea.The theorem(and a letter
                                                dicularto the:r-axis(Fig. 5.12).
                     from Galileo) were
of recommendation
enoughto win Cavalied a chai at the
Univelsityof Bolognain 1629.The solid
                                                                                          2lt -7
geometryversionin Example 3, which
Cavalieri neverproved,was given his name
by later geometen.




                                   have
                     Crcsssections
                     the samelongthat
                     everypoint in [d, b]




                                                5-t2 The wedgeof Example slicedperpendicular
                                                                          2.
                                                                     sections redangles.
                                                                             are
                                                to the x-axis.
                                                             The cross



                                                Step 2: Theformula for A(x). The cross section at r is a rectangleof area

                                                                   A(') : (heishtxwidth) (zJe - 'r)
                                                                                     : (r)
                                                                                           : z x t t: Y - x ' .
                                                                                               ^,

                                                Step 3: The linxits of integration. The rectangles from r :0                          to.r :3.
                                                                                                 run
                                                Step 4: The volune.
                                                                             ?b            ^3
                                                                                  Ax)dx: | 2tcJ9x2dx
                                                                                                -
                                                                      v:l
                                                                            Ja            JO

                                                                                 113
                                                                        : -;e - xr3/'
                                                                                   )o
      I




   ,f                                                                   : o+?e),/'             1! =          lf r1r. nncsfalc-
                                                                            J                  . L n d s u b s t i l U r eb r c k .

                                                                                                                                                 tr
                                                                        -   19



                                                                  Cavalieri's Theorerrr
                                                EXAMPLE 3

                                                Cavalieri's theoremsaysthat solids with equal altitudesand identical parallel cross-
               theorem. Thesesolids
5.13 Cavalieri's
                                                section areashave the samevolume (Fig. 5.13). We can seethis imrnediately from
havethe samevolume.You can illustrate
                                                Eq. (1) becauset}Ie cross-sectionarea function A(.r) is the same in each case. f
this yourself
            with stacks coins.
                      of
Exercises
                                                                                                                      5.2       377




Exercises
        5.2
            Areas
Cross-Section
In Exercises and 2, find a fomula for the arcaA(r) of the cross
             I
sectionsof the solid perpendicular the l-axrs.
                                 to
 1. The solid lies betweenplanesperpendicular the r-axis at i =
                                             to
    -l and,r:1. In eachcase,the crosssections     perpendicular
    to the r-axis betweentheseplanesrun from the semicircley =
    -^4 - ,r2 to the semicircley : 4[ - ,2.
   a)   The cross sectionsme circular disks with diametersin the
        xy-plane.
                                                                                            planesperpendicular ther-axis at.r : 0
                                                                       The solidliesbetween                    to
                                                                       andl : 4. The crosssectionsperpendicular the-t-axisbetween
                                                                                                               to
                                                                       these planes run from the parabola y = -aE to the parabola
                                                                       r: Ji.
                                                                       a)   The cross sectionsare circular disks with diametersin the
                                                                            r)-plane.




   b)   The crcsssections squares
                        are     with bases ther),-plane.
                                         in




                                                                       b)   The crosssectionsare squareswith basesin the r)-plane.




   c)   The cross sectionsarc squarcswith diagonalsin the xy-
        plane. (The length of a square'sdiagonalis .,4 times the
        lengthof its sides.)


                                                                       c)   The crosssections squares
                                                                                            are         with diagonals ther)-plane.
                                                                                                                      in
                                                                       d)   The crosssectionsare equilateraltriangleswith basesin the
                                                                            rJ-plane.

                                                                   Volumes Slicing
                                                                          by
                                                                   Find the volumesof the solids in Exercises3-12.
                                                                    3. The solid lies betweenplanesperpendicular the,-axis at .r : 0
                                                                                                               to
                                                                       and r :4. The crosssections  perpendicular the axis on the
                                                                                                                  to
        The crosssectionsarc equilateraltriangleswith bases the
                                                           in          interval 0 5 i :4 are squareswhose diagonalsrun ftom the
   d)
                                                                       parabola1 : -.vE to the parabolaf : Jtr.
        ry-plane.
374      chapter5: Applications Integrals
                              of

 4, The solid lies between planes perpendicularto the r-axis at -x :
    - 1 and r : I . The cross sections perpendicular to the ir axis are
                                    run from lhe parabolay - x7 to
    circulardisks whosediamerers
    the Darabola :2 - t2.
                  t




 5. The solid lies betweenplanesperpendicular the r-axis at
                                             to                           Cavalieri's
                                                                                    Theorem
    r : - I andr : 1. The crosssections
                                       perpendicular the axis
                                                  to
                                                                          ll. A twisted solid. A square sidelengths lies in a planeper-
                                                                                                        of
    betweentheseplanesare vertical squares whosebase edges
                                                                              pendicular a line L Onevertex the square on t. As this
                                                                                        to                   of           lies
    run from the semicircle - - 1(1- rt lo rhe semicircle -
                                                         y
                            J
                                                                              squ&emovesa distance alongl, the square
                                                                                                     l?                     tums one revo-
    !7-7.
                                                                              lution aboutZ to generatea corkscrewlike column with square
                                perpendicular ther-axis at r =
 6. The solidlies between planes           to                                 crosssections.
      I and n: l. The crosssections  perpendicular the ,r-axis
                                                 to
                                                                              a)   Find the volume of the column.
                  planes squares
    between these        are       whosediagonals frcm the
                                                  run
                                                                              b)   What will the volumebe if the squaretums twice instead
    semicircle : -{     -7 to the semicircle
                                           y: JT=V. Qn
              )                                                                             Givercasons your answer
                                                                                   of once?              for
    lengthof a square's
                      diagonal r/Z timesthelengthof its sides.)
                               is
                                     berween curve) : 2 vfi;                                        planes
                                                                          12. A solid lies between
 7. t he base thesolidis thereSion
             of                             lhe
                                        The qoss sections                     perpendicular tbe -{-axisat
    and the interval[0, r] on the.r-axis.               perpen-                              to
                                                                              -r = 0 ard .r : 12.The cross
    dicularto the r-axis are
                                                                              sections planes    perpendicular
                                                                                       by
                                                                              to ther-axis arecirculardisks
                                                                              whosediametersrun from the line
                                                                              ) : -r/2 to the line ) : -r.
                                                                              Explain why the solid has the
                                                                              same  volumeas a right circular
                                                                              conewith baseradius3 and
                                                                              height12.
                                                                          13. Cavalieri'soriginal theorem. ProveCavalieri's
                                                                                                                          original theo
         veftical equilateral triangles with bases running from the
    a)
                                                                             rem (marginalnote, page 376), assuming
                                                                                                                  that each region is
         I-axrs to me culve:
                                                                             boundedabove belowby thegraphs continuous
                                                                                         and                   of          functions.
    b)   vertical squareswith basesrunning from the r-axis to the
                                                                          t4. The volume of a hemisphere(a classical
         cutve.                                                                                                    application af Cav-
                                                                             alieri's theorem). Derive the formula V : (2/3)zR3 for the
 8. The solid lies between planes pe.pendicularto the r-axis at -x =
                                                                             volumeof a hemisphere radiusR by comparing crosssec-
                                                                                                    of                       its
    -r /3 and x : xr/3. The crcss sectionsperpendicularto the
                                                                  r-
                                                                             tionswith the crosssections a solid right circularcylinderof
                                                                                                         of
    axis are
                                                                             mdiusR andheightR from which a solidright circularconeof
         circular disks with diameters running from the curve y :
    a)
                                                                             baseradius andheishtR hasbeenremoved.
                                                                                         R
         tan,r to the curve ): secr;
         verlical squares whose base edges run from the curve y :
    b)
         tanr to the cuNe ): SeCr.
 9. The solid lies betweenplanesperpendicularto the y-axis at y : 0
    and ) : 2. The cross sectionsperpendicularto the ]-axis are cir-
    cular disks with diametersrunning from they-axis to the parabola


10. The base of the solid is the disk j'? + ),2 5 1. The cross sections
                                                           I and y - I
    by planesperpendicularto the )-axis between], -
    are isoscelesdght triangles with one leg in the disk.
5.3 Volumesof Solidsof Revolution-Disksand Washers                 379




                                                              rn::rt sotids Revorution-Disks
                                                                          of
                                    E                n
                                                     The most common applicarion of the method of slicing is to solids of revolution.
                                                     Solids of revolution are solids whose shapescan be generatedby revoJvingplane
                                                     regions about axes. Thread spools are solids of revolution; so are hand weights
                                                     and billiard balls. Solids of revolutign sometimeshave volumes we can find with
                                                     formulas liom geometry, as in the case of a billiard ball. But when we want to
                                                     find the volume of a blimp or to predict the weight of a part we are going to have
                                                     turned on a lathe, formulas from geometry are of little help and we tum to calculus
                                                     lor lhe un:* ers.




                                                                                  6
                                                                                    revolution

                                                                                                                        Axisof
                                                 revolution


                                                                                                                         s
                                                 #

                                                          If we can arange for the region to be the region betweenthe graph of a contin-
                                                     uous function I = R(r), a 5 x < ,, and the x-axis, and for the axis of revolution
                                                     to be the x-axis (Fig. 5.14), we can find the solid's volume in the following way.
                                                          The typical cross section of the solid perpendicularto the axis of revolution is
                                                     a disk of radius R(x) and area

                                                                              A(.r) : 71136iut1: rlR(x)12.
                                                                                               :

                                                     The solid's volume, being the integral ofA from.r - a to x : b, is the integral of
               Cross sectionperpendjcularto
                                                     zfR(x)1'zfrom a to D.
                the axis at r is a disk ofarea
               ,4(r) = 7r(radius)2 tt (R(,r))2
                                    =



                                                         Volume of a Solid of Revolution (Rotation About the .r-axis)

                                                        The volume of the solid generatedby revolving about the.{-axis the region
                                                        betweenthe -r-axis and the graph of the continuousfunction ) : R(r), a S
                                                        :r 5 b, is

                                                                        v = [ , ftua;ur1t : [' r l R 1 x ) l 2 l r .
                                                                                        a*                                        (1)
                                                                                                              z
                                                                            J.               J,,



                                                     EXAMPLE I         The region betweenthe curye y :.,rE,O <.{ S 4, and the r-axis
and the x-axisfrom a to b about the
x-axis.                                              is revolved about the .x axis to generatea solid. Find its volume.
380           Chapter Applications lntegrals
                    5:           of

   :l,
                                           Solution We draw ligures showing the region, a typical radius, and the generated
                                           solid (Fig. 5.15).The volumeis
                                                                       fb
                                                              v : I trlR(x)'dx                                          F - qr l r


                                                                       f1
                                                                 : I rlrtTax                                             /l1ft        i
                                                                   Jo-
                                                                                                         (4t2
                                                                           f4
                                                                                   - u . ,t 2 l a
                                                                 =tr
                                                                          lo x d x         LJo =z-:8n.
                                                                                                1
                                                                                                                                           l
                                                                         J


         ,,    .f

                                               How to Find Volumes Using Eq. (1)
                                               1, Draw the regionandidentifythe radiusfunctionR(x).
                                               2. Square R(r) andmultiply by z.
                                               3. Intesrate find the volume.
                                                          to



                                               The axis of revolution the next example not the .x-axis, the rule for
                                                                     in                is             but
                                           calculating volumeis thesame:
                                                     the                  Integmtelr(radius)2
                                                                                            betweenappropriate
                                                                                                             limits.
                       (b)

5.15 The region(a) and solid(b) in         EXAMPLE2     Find the volumeof the solid generated revolvingthe region
                                                                                            by
Exampl1.e
                                           bounded y : .rf andthe lines) : l, x:4 about line y : 1.
                                                 by                                      the
                                           Solution We drawfiguresshowing region,a typicalradius,andthe generated
                                                                          the
                                           solid (Fig. 5.16).The volumeis
                                                                         F
                                                                                                            f( lrr
                                                                V:      I r l 'R ( x ) 1 d r
                                                                                         z
                                                                        J,
                                                                            r1
                                                                   : I n l' J x _ t ) - d x
                                                                     Jt

                                                                                 f4
                                                                   : o I 'l , z  E+ r ) a ,
                                                                       Jt
                                                                                          ^             .4
                                                                                      ^ tt                     tn
                                                                             lx-               - + x l : -I
                                                                   :Tl                2':x
                                                                               ^
                                                                                          J                     o
                                                                             Lz                        lr                                  -j
                                                To find the volume of a solid generated by revolving a region between the
                                           )-axis and a curve x : R()), c < l : d, about the y-axis, we use Eq. (i) with r
                                           replaced by ).




                                               Volume of a Solid of Revolution (Rotation About the y-axis)


                      (b)

5.16 The region(a) and solid(b) in
Example2.




                                                                                                                     -
5.3 Volumesof Solidsof Revolution_Disksand Washers                               391


                                     EXAMPLE 3       Find the volume of the solid generatedby revolving Lnereglon
                                     betweenthe y-axis and the curve x:2/y,I   S l,:4, aboutthe y-axis.
                                     Solution We draw figures showing the region, a typical radius, and the generated
                                     solid (Fig. 5.17).The volumeis

                                                        v:                :zln6)1'z
                                                                                ay                     l_ l (lr
                                                                                                         r
                                                                l,o
                                                           = ;) o,
                                                                t4         /'r2

                                                            J,                                         /11 I
                                                                                                          r

                                                                     r^ 4                     T r'1.
                                                                                          l3l
                                                                 I - tlt : qr | -: | : 4n r . t
                                                           -t
                                                                 Jt v         L )lr       L4J
                                                                                                                                         .l


                                     EXAMPLE 4      Find the volume of the solid generatedby revolving the region
                                     betweenthe parabola :12 + I and the line I :3 aboutthe line; :3.
                                                        I

                                     so/ut on We draw ligures showing the region, a typical radius, and the generated
                                     solid (Fig. 5.18). The volume is
(                                                                          N1
                                                                ,:                                                    l.r r: )
                                                                          J nrlR(y)'?dy
                                                                            pD
                                                                                                                      /11r:1    rf -ll
                                                                               rl2 - y212
                                                                     :                  dy
                                                                           I
               (b)
                                                                                 /1
                                                                     :r
5.17 The region(a) and solid(b) in                                                    L4-4y2+y4ldy
                                                                                I
Example3.
                                                                                            .,,51f
                                                                     : 7 r | 4 y - ; - y '+ a I
                                                                                   4
                                                                           l       J            )l^
                                                                            I

                                                                          64rJ2
                                                                            t5


                                             R(])=3-()'?+ 1)
                                         )

                                                                                        z

                                                                                         0

                                                                                       -.'t

                                                                                                                        r:t-+r
                                                                                                                  I
                                                         (a.)                                                  (b)
5.78 The region(a) and solid(b) in
Example4.
                                                                                                                                     l
Chapter 5: Applicationsof lntegrals
382


                                            The WasherMethod
                                            If the region we revolve to generate a solid does not border on or cross the axis of
                                            revolution, the solid has a hole in it (Fig. 5.19). The cross sectionsperpendicular
                                            to the axis of revolution ate washersinstead of disks. The dimensionsof a typical
                                            washer are
                                                                                              R(-r)
                                                                          Outerradius:
      ,                                                                                      rx )
                                                                          Inner radius:
                                                       areais
                                            The washer's
                                                         A(x) : TtlR(;)lz rVG)12: o (tn{)l' - tt(r)lt).
                                                                        -



                                               The Washer Fonnula for Finding Volumes
                                                                           fb
                                                                                           .lrtx)f)dx
                                                                    v: I                                                 (3)
                                                                       ra  ( [ R r x ) | ' : |
                                                                                  |
                                                                                     tl
                                                                                 outer        inner
                                                                                             radius
                                                                                 mdius
                                                                                squarcd     squarcd
              ) = R(-r)


                                            Notice that the function integrated in Eq. (3) is t(Rz - r2), not T (R - r)'z. Also
                                            notice that Eq. (3) gives the disk method fomula if /(jr) is zero throughout [a, r].
                                            Thus. the disk method is a soecial case of the washer method.


                                                            The region boundedby the curve y : xz + | and the line y :
                                            EXAMPLE 5
                                            -x t 3 is revolved about the -r-axis to generatea solid. Find the volume of the
                                            solid.
5.?9 The crosssectionsof the solid of
revolution generated here are washers,      Solution
not disks, the integralf Ak) dx leads
          so
                                            Sfep t; Draw the region and sketcha line segmentacrossit perpendicularto the
to a slightlydifferentformula.
                                                                (the red segment Fig. 5.20).
                                                                                  in
                                            a;dsof revolution
                                            Step 2,' Find the limits of integrationby finding the.r-coordinates the intersection
                                                                                                              of
                                            points.
                                                                                 x2+l:-x+3
                                                                           ,'+*-2:o
                                                                         (x+2)(x-l):0
                                                                            , -_1           .-l

                                            Step 3; Find the outer and inner radii of the washerthat would be swept out by
                                            the line segmentif it were revolved about the x-axis along with the region. ftVe
                                            drewthe washer Fig. 5.21,but in your own work you neednot do that.)These
                                                              in
                                            radii are the distances the endsof the line sesmentfrom the axis of revolution.
                                                                   of
                                                                                          R(x) : -1 -1-
                                                                                                      3
                                                                     Outer radius:
                                                                                          r(x):a241
                                                                     Inner radius:
5.3 Volumes Solids Revolution-Disks Washers 383
                                                                                      of     of               and




                                                                           +-1


                                     (1,2)




        lntegratlon

j . . i T h er e g i o ni n E x a m p l 5 s p a n n e d y
                                        e             b
a line segmentperpendicular the axis     to
of revolution.      when the regionis                                                                        5.21 The inner and outer radii of the
                                                                   cmss section
                                                            Washer
            about the x-axis, line
revolved                           the                      Outerradius: = r+3                               washersweptout by the line segmentin
                                                                       R(r)
            will generatea washer.
segment                                                                r(.r) = 12 + I
                                                            Innerradius:                                     Fi9.5.20.



                                                            Step 4; Evaluate the volume integral.
                                                                                             rb
                                                                                                          -
                                                                                  ,=                ([Rrx)l' lr(x)]')
                                                                                                                    dx                  F ( t 1 - il
                                                                                            J,,
                                                                                             lt              ^
                                                                                        =           ({ x | 3)'    tx' I lt'tdt
                                                                                                                                        I ltn!1 l
                                                                                            /_rtr
                                                                                             fl
                                                                                        = / z(S         6x   x2    xa;dx
                                                                                          J-2

                                                                                                             r'    r'l'          tllr
                                                                                        -'l*t^
                                                                                                             i      sl,           s                    LI




                                                               How to FindVolumesby the WasherMethod
                                                                    Draw the region and sketch a line segmentacross it perpend.icularto
                                                               l.
                                                                    the q,is of revolution. When the region is revolved, this segmentwill
                                                                    generatea typical washer cross section of the generatedsolid.
                                                               2.   Find the limils of integration.
                                                                    Find the outer qnd inner rqdii of the washer swept out by the line
                                                                    segment.
                                                               4.   Integrate to flnd the volume.



                                                                To find the volume of a solid generatedby revolving a region about the )-axis,
                                                            we use the stepslisted above but integratewith respectto l instead of ,r.


                                                                               The regionboundedby the parabola : x2 and the line y : 2y
                                                            EXAMPLE 6                                                I
                                                                         quadrantis revolved about the )-axis to generatea solid. Find the volume
                                                            in the first
                                                            of the solid.
384    Chapter Applications Integrals
             5:           of

                                        Solution
                                        Step t.' Draw the region and sketch a line segment across it perpendicular to the
                                        axis of revolution, in this case the ),-axis (Fig. 5.22).
                                        Step 2: The line and parabola intersect at ) : 0 and l : 4, so the limits of inte-
                                        grationare c :0 andd :4.
                                        Step 3.' The radii of the washer swept out by the line segment are R(y) : a/ry,
                                        r(y) : y/2 (Figs.5.22 and 5.23).




                                        ?




t22 The region,limitsof integration,                                                523 The washersweptout by the line
and radii in Example
                   6.                                                               segmentin Fig.5.22.


                                       Step 4:
                                                                                                                Eq. (-l ) 'irh i
                                                                        -
                                                        ,:     , (tnci)l, tr(y)t2)
                                                                                dy
                                                             1
                                                          : l,^ -ltrJ')r,
                                                               (ltl'                                            sleps I .rnd -l


                                                          :L'(,-';)*:l+-i]:,                         8
                                                                                                       3


                                       EXAMPLE 7          The region in the first quadrantenclosedby the parabola y :2e2,
                                       the y-axis, and the line ) : I is revolved about the line,r : 3/2 to generatea solid.
                      )=*2or*:f
                          t
 5ol
                                       Find the volume of the solid.
                       4!.=;-ti
                                       Solution
                                       Step 1.' Draw the region and sketch a line segment across it perpendicular to the
Er
                                       axis of revolution, in this case the line ; :3/2 (Fig. 5.24).
                                       Step 2: Tbe limits of integration are ) : 0 to l : 1.
                                       Step 3.' The radii of the washer swept out by the line segment arc R(y) :3/2,
524 The region,limitsof integration,
                                       r(y) : (3/2) - -/1, (Fiss. 5.24 and 5.25).
and radii in Example
                   7.
Chapter 5 Thomas 9 Ed F 2008
Chapter 5 Thomas 9 Ed F 2008
Chapter 5 Thomas 9 Ed F 2008
Chapter 5 Thomas 9 Ed F 2008
Chapter 5 Thomas 9 Ed F 2008
Chapter 5 Thomas 9 Ed F 2008
Chapter 5 Thomas 9 Ed F 2008
Chapter 5 Thomas 9 Ed F 2008
Chapter 5 Thomas 9 Ed F 2008
Chapter 5 Thomas 9 Ed F 2008
Chapter 5 Thomas 9 Ed F 2008
Chapter 5 Thomas 9 Ed F 2008
Chapter 5 Thomas 9 Ed F 2008
Chapter 5 Thomas 9 Ed F 2008
Chapter 5 Thomas 9 Ed F 2008
Chapter 5 Thomas 9 Ed F 2008
Chapter 5 Thomas 9 Ed F 2008
Chapter 5 Thomas 9 Ed F 2008
Chapter 5 Thomas 9 Ed F 2008
Chapter 5 Thomas 9 Ed F 2008
Chapter 5 Thomas 9 Ed F 2008
Chapter 5 Thomas 9 Ed F 2008
Chapter 5 Thomas 9 Ed F 2008
Chapter 5 Thomas 9 Ed F 2008
Chapter 5 Thomas 9 Ed F 2008
Chapter 5 Thomas 9 Ed F 2008
Chapter 5 Thomas 9 Ed F 2008
Chapter 5 Thomas 9 Ed F 2008
Chapter 5 Thomas 9 Ed F 2008
Chapter 5 Thomas 9 Ed F 2008
Chapter 5 Thomas 9 Ed F 2008
Chapter 5 Thomas 9 Ed F 2008
Chapter 5 Thomas 9 Ed F 2008
Chapter 5 Thomas 9 Ed F 2008
Chapter 5 Thomas 9 Ed F 2008
Chapter 5 Thomas 9 Ed F 2008
Chapter 5 Thomas 9 Ed F 2008
Chapter 5 Thomas 9 Ed F 2008
Chapter 5 Thomas 9 Ed F 2008
Chapter 5 Thomas 9 Ed F 2008
Chapter 5 Thomas 9 Ed F 2008
Chapter 5 Thomas 9 Ed F 2008
Chapter 5 Thomas 9 Ed F 2008
Chapter 5 Thomas 9 Ed F 2008
Chapter 5 Thomas 9 Ed F 2008
Chapter 5 Thomas 9 Ed F 2008
Chapter 5 Thomas 9 Ed F 2008
Chapter 5 Thomas 9 Ed F 2008
Chapter 5 Thomas 9 Ed F 2008
Chapter 5 Thomas 9 Ed F 2008
Chapter 5 Thomas 9 Ed F 2008
Chapter 5 Thomas 9 Ed F 2008
Chapter 5 Thomas 9 Ed F 2008
Chapter 5 Thomas 9 Ed F 2008
Chapter 5 Thomas 9 Ed F 2008
Chapter 5 Thomas 9 Ed F 2008
Chapter 5 Thomas 9 Ed F 2008
Chapter 5 Thomas 9 Ed F 2008
Chapter 5 Thomas 9 Ed F 2008
Chapter 5 Thomas 9 Ed F 2008
Chapter 5 Thomas 9 Ed F 2008

Contenu connexe

Tendances

10. polar coordinates x
10. polar coordinates x10. polar coordinates x
10. polar coordinates xharbormath240
 
Modern features-part-4-evaluation
Modern features-part-4-evaluationModern features-part-4-evaluation
Modern features-part-4-evaluationzukun
 
Introduction To Polar Coordinates And Graphs
Introduction To Polar Coordinates And GraphsIntroduction To Polar Coordinates And Graphs
Introduction To Polar Coordinates And Graphseekeeney
 
1 polar coordinates
1 polar coordinates1 polar coordinates
1 polar coordinatesmath267
 
THE QUALITATIVE ANALYZE OF THE CARTOGRAPHICAL PROJECTIONS USED IN ROMANIA
THE QUALITATIVE ANALYZE OF THE CARTOGRAPHICAL PROJECTIONS USED IN ROMANIATHE QUALITATIVE ANALYZE OF THE CARTOGRAPHICAL PROJECTIONS USED IN ROMANIA
THE QUALITATIVE ANALYZE OF THE CARTOGRAPHICAL PROJECTIONS USED IN ROMANIAmihai_herbei
 
36 area in polar coordinate
36 area in polar coordinate36 area in polar coordinate
36 area in polar coordinatemath266
 
34 polar coordinate and equations
34 polar coordinate and equations34 polar coordinate and equations
34 polar coordinate and equationsmath266
 
02 representing position and orientation
02 representing position and orientation02 representing position and orientation
02 representing position and orientationcairo university
 
Section 11 1-notes_2
Section 11 1-notes_2Section 11 1-notes_2
Section 11 1-notes_2kerrynix
 
11. polar equations and graphs x
11. polar equations and graphs x11. polar equations and graphs x
11. polar equations and graphs xharbormath240
 
20110319 parameterized algorithms_fomin_lecture03-04
20110319 parameterized algorithms_fomin_lecture03-0420110319 parameterized algorithms_fomin_lecture03-04
20110319 parameterized algorithms_fomin_lecture03-04Computer Science Club
 

Tendances (19)

Venn diagram
Venn diagramVenn diagram
Venn diagram
 
Polar coordinates
Polar coordinatesPolar coordinates
Polar coordinates
 
10. polar coordinates x
10. polar coordinates x10. polar coordinates x
10. polar coordinates x
 
Modern features-part-4-evaluation
Modern features-part-4-evaluationModern features-part-4-evaluation
Modern features-part-4-evaluation
 
[Download] rev chapter-9-june26th
[Download] rev chapter-9-june26th[Download] rev chapter-9-june26th
[Download] rev chapter-9-june26th
 
Introduction To Polar Coordinates And Graphs
Introduction To Polar Coordinates And GraphsIntroduction To Polar Coordinates And Graphs
Introduction To Polar Coordinates And Graphs
 
1 polar coordinates
1 polar coordinates1 polar coordinates
1 polar coordinates
 
Chapter 4 98
Chapter 4 98Chapter 4 98
Chapter 4 98
 
Pc 10.7 notes_polar
Pc 10.7 notes_polarPc 10.7 notes_polar
Pc 10.7 notes_polar
 
THE QUALITATIVE ANALYZE OF THE CARTOGRAPHICAL PROJECTIONS USED IN ROMANIA
THE QUALITATIVE ANALYZE OF THE CARTOGRAPHICAL PROJECTIONS USED IN ROMANIATHE QUALITATIVE ANALYZE OF THE CARTOGRAPHICAL PROJECTIONS USED IN ROMANIA
THE QUALITATIVE ANALYZE OF THE CARTOGRAPHICAL PROJECTIONS USED IN ROMANIA
 
125 7.3 and 7.5
125 7.3 and 7.5125 7.3 and 7.5
125 7.3 and 7.5
 
Curves part two
Curves part twoCurves part two
Curves part two
 
Kernel Methods on Manifolds
Kernel Methods on ManifoldsKernel Methods on Manifolds
Kernel Methods on Manifolds
 
36 area in polar coordinate
36 area in polar coordinate36 area in polar coordinate
36 area in polar coordinate
 
34 polar coordinate and equations
34 polar coordinate and equations34 polar coordinate and equations
34 polar coordinate and equations
 
02 representing position and orientation
02 representing position and orientation02 representing position and orientation
02 representing position and orientation
 
Section 11 1-notes_2
Section 11 1-notes_2Section 11 1-notes_2
Section 11 1-notes_2
 
11. polar equations and graphs x
11. polar equations and graphs x11. polar equations and graphs x
11. polar equations and graphs x
 
20110319 parameterized algorithms_fomin_lecture03-04
20110319 parameterized algorithms_fomin_lecture03-0420110319 parameterized algorithms_fomin_lecture03-04
20110319 parameterized algorithms_fomin_lecture03-04
 

Similaire à Chapter 5 Thomas 9 Ed F 2008

Newton cotes integration method
Newton cotes integration  methodNewton cotes integration  method
Newton cotes integration methodshashikant pabari
 
Hormann.2001.TPI.pdf
Hormann.2001.TPI.pdfHormann.2001.TPI.pdf
Hormann.2001.TPI.pdfssuserbe139c
 
Purdue Prelecture Content IMPACT
Purdue Prelecture Content IMPACTPurdue Prelecture Content IMPACT
Purdue Prelecture Content IMPACTdoylejackd
 
Application Of vector Integration and all
Application Of vector Integration and allApplication Of vector Integration and all
Application Of vector Integration and allMalikUmarKhakh
 
Master Thesis on the Mathematial Analysis of Neural Networks
Master Thesis on the Mathematial Analysis of Neural NetworksMaster Thesis on the Mathematial Analysis of Neural Networks
Master Thesis on the Mathematial Analysis of Neural NetworksAlina Leidinger
 
Dumitru Vulcanov - Numerical simulations with Ricci flow, an overview and cos...
Dumitru Vulcanov - Numerical simulations with Ricci flow, an overview and cos...Dumitru Vulcanov - Numerical simulations with Ricci flow, an overview and cos...
Dumitru Vulcanov - Numerical simulations with Ricci flow, an overview and cos...SEENET-MTP
 
Formulas for Surface Weighted Numbers on Graph
Formulas for Surface Weighted Numbers on GraphFormulas for Surface Weighted Numbers on Graph
Formulas for Surface Weighted Numbers on Graphijtsrd
 
Double_Integral.pdf
Double_Integral.pdfDouble_Integral.pdf
Double_Integral.pdfd00a7ece
 
Parallel Evaluation of Multi-Semi-Joins
Parallel Evaluation of Multi-Semi-JoinsParallel Evaluation of Multi-Semi-Joins
Parallel Evaluation of Multi-Semi-JoinsJonny Daenen
 
image segmentation image segmentation.pptx
image segmentation image segmentation.pptximage segmentation image segmentation.pptx
image segmentation image segmentation.pptxNaveenKumar5162
 
Electromagnetic theory EMT lecture 1
Electromagnetic theory EMT lecture 1Electromagnetic theory EMT lecture 1
Electromagnetic theory EMT lecture 1Ali Farooq
 
Real meaning of functions
Real meaning of functionsReal meaning of functions
Real meaning of functionsTarun Gehlot
 
Geometric Applications of the Definite Integrals- Sampayan, Jill Ann.pptx
Geometric Applications of the Definite Integrals- Sampayan, Jill Ann.pptxGeometric Applications of the Definite Integrals- Sampayan, Jill Ann.pptx
Geometric Applications of the Definite Integrals- Sampayan, Jill Ann.pptxGtScarlet
 
Data exploration and graphics with R
Data exploration and graphics with RData exploration and graphics with R
Data exploration and graphics with RAlberto Labarga
 

Similaire à Chapter 5 Thomas 9 Ed F 2008 (20)

Chapter 4
Chapter 4Chapter 4
Chapter 4
 
Newton cotes integration method
Newton cotes integration  methodNewton cotes integration  method
Newton cotes integration method
 
Hormann.2001.TPI.pdf
Hormann.2001.TPI.pdfHormann.2001.TPI.pdf
Hormann.2001.TPI.pdf
 
Purdue Prelecture Content IMPACT
Purdue Prelecture Content IMPACTPurdue Prelecture Content IMPACT
Purdue Prelecture Content IMPACT
 
Differential calculus
Differential calculusDifferential calculus
Differential calculus
 
Application Of vector Integration and all
Application Of vector Integration and allApplication Of vector Integration and all
Application Of vector Integration and all
 
Master Thesis on the Mathematial Analysis of Neural Networks
Master Thesis on the Mathematial Analysis of Neural NetworksMaster Thesis on the Mathematial Analysis of Neural Networks
Master Thesis on the Mathematial Analysis of Neural Networks
 
10. functions
10. functions10. functions
10. functions
 
Calculus I basic concepts
Calculus I basic conceptsCalculus I basic concepts
Calculus I basic concepts
 
Dumitru Vulcanov - Numerical simulations with Ricci flow, an overview and cos...
Dumitru Vulcanov - Numerical simulations with Ricci flow, an overview and cos...Dumitru Vulcanov - Numerical simulations with Ricci flow, an overview and cos...
Dumitru Vulcanov - Numerical simulations with Ricci flow, an overview and cos...
 
Chatper 15
Chatper 15Chatper 15
Chatper 15
 
Formulas for Surface Weighted Numbers on Graph
Formulas for Surface Weighted Numbers on GraphFormulas for Surface Weighted Numbers on Graph
Formulas for Surface Weighted Numbers on Graph
 
Double_Integral.pdf
Double_Integral.pdfDouble_Integral.pdf
Double_Integral.pdf
 
Parallel Evaluation of Multi-Semi-Joins
Parallel Evaluation of Multi-Semi-JoinsParallel Evaluation of Multi-Semi-Joins
Parallel Evaluation of Multi-Semi-Joins
 
image segmentation image segmentation.pptx
image segmentation image segmentation.pptximage segmentation image segmentation.pptx
image segmentation image segmentation.pptx
 
Electromagnetic theory EMT lecture 1
Electromagnetic theory EMT lecture 1Electromagnetic theory EMT lecture 1
Electromagnetic theory EMT lecture 1
 
Real meaning of functions
Real meaning of functionsReal meaning of functions
Real meaning of functions
 
Geometric Applications of the Definite Integrals- Sampayan, Jill Ann.pptx
Geometric Applications of the Definite Integrals- Sampayan, Jill Ann.pptxGeometric Applications of the Definite Integrals- Sampayan, Jill Ann.pptx
Geometric Applications of the Definite Integrals- Sampayan, Jill Ann.pptx
 
Data exploration and graphics with R
Data exploration and graphics with RData exploration and graphics with R
Data exploration and graphics with R
 
BS01_10900_AE_C.pdf
BS01_10900_AE_C.pdfBS01_10900_AE_C.pdf
BS01_10900_AE_C.pdf
 

Dernier

The Fit for Passkeys for Employee and Consumer Sign-ins: FIDO Paris Seminar.pptx
The Fit for Passkeys for Employee and Consumer Sign-ins: FIDO Paris Seminar.pptxThe Fit for Passkeys for Employee and Consumer Sign-ins: FIDO Paris Seminar.pptx
The Fit for Passkeys for Employee and Consumer Sign-ins: FIDO Paris Seminar.pptxLoriGlavin3
 
Ryan Mahoney - Will Artificial Intelligence Replace Real Estate Agents
Ryan Mahoney - Will Artificial Intelligence Replace Real Estate AgentsRyan Mahoney - Will Artificial Intelligence Replace Real Estate Agents
Ryan Mahoney - Will Artificial Intelligence Replace Real Estate AgentsRyan Mahoney
 
The Role of FIDO in a Cyber Secure Netherlands: FIDO Paris Seminar.pptx
The Role of FIDO in a Cyber Secure Netherlands: FIDO Paris Seminar.pptxThe Role of FIDO in a Cyber Secure Netherlands: FIDO Paris Seminar.pptx
The Role of FIDO in a Cyber Secure Netherlands: FIDO Paris Seminar.pptxLoriGlavin3
 
A Deep Dive on Passkeys: FIDO Paris Seminar.pptx
A Deep Dive on Passkeys: FIDO Paris Seminar.pptxA Deep Dive on Passkeys: FIDO Paris Seminar.pptx
A Deep Dive on Passkeys: FIDO Paris Seminar.pptxLoriGlavin3
 
Dev Dives: Streamline document processing with UiPath Studio Web
Dev Dives: Streamline document processing with UiPath Studio WebDev Dives: Streamline document processing with UiPath Studio Web
Dev Dives: Streamline document processing with UiPath Studio WebUiPathCommunity
 
TrustArc Webinar - How to Build Consumer Trust Through Data Privacy
TrustArc Webinar - How to Build Consumer Trust Through Data PrivacyTrustArc Webinar - How to Build Consumer Trust Through Data Privacy
TrustArc Webinar - How to Build Consumer Trust Through Data PrivacyTrustArc
 
The Ultimate Guide to Choosing WordPress Pros and Cons
The Ultimate Guide to Choosing WordPress Pros and ConsThe Ultimate Guide to Choosing WordPress Pros and Cons
The Ultimate Guide to Choosing WordPress Pros and ConsPixlogix Infotech
 
Ensuring Technical Readiness For Copilot in Microsoft 365
Ensuring Technical Readiness For Copilot in Microsoft 365Ensuring Technical Readiness For Copilot in Microsoft 365
Ensuring Technical Readiness For Copilot in Microsoft 3652toLead Limited
 
Anypoint Exchange: It’s Not Just a Repo!
Anypoint Exchange: It’s Not Just a Repo!Anypoint Exchange: It’s Not Just a Repo!
Anypoint Exchange: It’s Not Just a Repo!Manik S Magar
 
"Debugging python applications inside k8s environment", Andrii Soldatenko
"Debugging python applications inside k8s environment", Andrii Soldatenko"Debugging python applications inside k8s environment", Andrii Soldatenko
"Debugging python applications inside k8s environment", Andrii SoldatenkoFwdays
 
Unraveling Multimodality with Large Language Models.pdf
Unraveling Multimodality with Large Language Models.pdfUnraveling Multimodality with Large Language Models.pdf
Unraveling Multimodality with Large Language Models.pdfAlex Barbosa Coqueiro
 
Nell’iperspazio con Rocket: il Framework Web di Rust!
Nell’iperspazio con Rocket: il Framework Web di Rust!Nell’iperspazio con Rocket: il Framework Web di Rust!
Nell’iperspazio con Rocket: il Framework Web di Rust!Commit University
 
A Journey Into the Emotions of Software Developers
A Journey Into the Emotions of Software DevelopersA Journey Into the Emotions of Software Developers
A Journey Into the Emotions of Software DevelopersNicole Novielli
 
Gen AI in Business - Global Trends Report 2024.pdf
Gen AI in Business - Global Trends Report 2024.pdfGen AI in Business - Global Trends Report 2024.pdf
Gen AI in Business - Global Trends Report 2024.pdfAddepto
 
How to write a Business Continuity Plan
How to write a Business Continuity PlanHow to write a Business Continuity Plan
How to write a Business Continuity PlanDatabarracks
 
Digital Identity is Under Attack: FIDO Paris Seminar.pptx
Digital Identity is Under Attack: FIDO Paris Seminar.pptxDigital Identity is Under Attack: FIDO Paris Seminar.pptx
Digital Identity is Under Attack: FIDO Paris Seminar.pptxLoriGlavin3
 
What's New in Teams Calling, Meetings and Devices March 2024
What's New in Teams Calling, Meetings and Devices March 2024What's New in Teams Calling, Meetings and Devices March 2024
What's New in Teams Calling, Meetings and Devices March 2024Stephanie Beckett
 
New from BookNet Canada for 2024: Loan Stars - Tech Forum 2024
New from BookNet Canada for 2024: Loan Stars - Tech Forum 2024New from BookNet Canada for 2024: Loan Stars - Tech Forum 2024
New from BookNet Canada for 2024: Loan Stars - Tech Forum 2024BookNet Canada
 
Transcript: New from BookNet Canada for 2024: BNC CataList - Tech Forum 2024
Transcript: New from BookNet Canada for 2024: BNC CataList - Tech Forum 2024Transcript: New from BookNet Canada for 2024: BNC CataList - Tech Forum 2024
Transcript: New from BookNet Canada for 2024: BNC CataList - Tech Forum 2024BookNet Canada
 
Transcript: New from BookNet Canada for 2024: Loan Stars - Tech Forum 2024
Transcript: New from BookNet Canada for 2024: Loan Stars - Tech Forum 2024Transcript: New from BookNet Canada for 2024: Loan Stars - Tech Forum 2024
Transcript: New from BookNet Canada for 2024: Loan Stars - Tech Forum 2024BookNet Canada
 

Dernier (20)

The Fit for Passkeys for Employee and Consumer Sign-ins: FIDO Paris Seminar.pptx
The Fit for Passkeys for Employee and Consumer Sign-ins: FIDO Paris Seminar.pptxThe Fit for Passkeys for Employee and Consumer Sign-ins: FIDO Paris Seminar.pptx
The Fit for Passkeys for Employee and Consumer Sign-ins: FIDO Paris Seminar.pptx
 
Ryan Mahoney - Will Artificial Intelligence Replace Real Estate Agents
Ryan Mahoney - Will Artificial Intelligence Replace Real Estate AgentsRyan Mahoney - Will Artificial Intelligence Replace Real Estate Agents
Ryan Mahoney - Will Artificial Intelligence Replace Real Estate Agents
 
The Role of FIDO in a Cyber Secure Netherlands: FIDO Paris Seminar.pptx
The Role of FIDO in a Cyber Secure Netherlands: FIDO Paris Seminar.pptxThe Role of FIDO in a Cyber Secure Netherlands: FIDO Paris Seminar.pptx
The Role of FIDO in a Cyber Secure Netherlands: FIDO Paris Seminar.pptx
 
A Deep Dive on Passkeys: FIDO Paris Seminar.pptx
A Deep Dive on Passkeys: FIDO Paris Seminar.pptxA Deep Dive on Passkeys: FIDO Paris Seminar.pptx
A Deep Dive on Passkeys: FIDO Paris Seminar.pptx
 
Dev Dives: Streamline document processing with UiPath Studio Web
Dev Dives: Streamline document processing with UiPath Studio WebDev Dives: Streamline document processing with UiPath Studio Web
Dev Dives: Streamline document processing with UiPath Studio Web
 
TrustArc Webinar - How to Build Consumer Trust Through Data Privacy
TrustArc Webinar - How to Build Consumer Trust Through Data PrivacyTrustArc Webinar - How to Build Consumer Trust Through Data Privacy
TrustArc Webinar - How to Build Consumer Trust Through Data Privacy
 
The Ultimate Guide to Choosing WordPress Pros and Cons
The Ultimate Guide to Choosing WordPress Pros and ConsThe Ultimate Guide to Choosing WordPress Pros and Cons
The Ultimate Guide to Choosing WordPress Pros and Cons
 
Ensuring Technical Readiness For Copilot in Microsoft 365
Ensuring Technical Readiness For Copilot in Microsoft 365Ensuring Technical Readiness For Copilot in Microsoft 365
Ensuring Technical Readiness For Copilot in Microsoft 365
 
Anypoint Exchange: It’s Not Just a Repo!
Anypoint Exchange: It’s Not Just a Repo!Anypoint Exchange: It’s Not Just a Repo!
Anypoint Exchange: It’s Not Just a Repo!
 
"Debugging python applications inside k8s environment", Andrii Soldatenko
"Debugging python applications inside k8s environment", Andrii Soldatenko"Debugging python applications inside k8s environment", Andrii Soldatenko
"Debugging python applications inside k8s environment", Andrii Soldatenko
 
Unraveling Multimodality with Large Language Models.pdf
Unraveling Multimodality with Large Language Models.pdfUnraveling Multimodality with Large Language Models.pdf
Unraveling Multimodality with Large Language Models.pdf
 
Nell’iperspazio con Rocket: il Framework Web di Rust!
Nell’iperspazio con Rocket: il Framework Web di Rust!Nell’iperspazio con Rocket: il Framework Web di Rust!
Nell’iperspazio con Rocket: il Framework Web di Rust!
 
A Journey Into the Emotions of Software Developers
A Journey Into the Emotions of Software DevelopersA Journey Into the Emotions of Software Developers
A Journey Into the Emotions of Software Developers
 
Gen AI in Business - Global Trends Report 2024.pdf
Gen AI in Business - Global Trends Report 2024.pdfGen AI in Business - Global Trends Report 2024.pdf
Gen AI in Business - Global Trends Report 2024.pdf
 
How to write a Business Continuity Plan
How to write a Business Continuity PlanHow to write a Business Continuity Plan
How to write a Business Continuity Plan
 
Digital Identity is Under Attack: FIDO Paris Seminar.pptx
Digital Identity is Under Attack: FIDO Paris Seminar.pptxDigital Identity is Under Attack: FIDO Paris Seminar.pptx
Digital Identity is Under Attack: FIDO Paris Seminar.pptx
 
What's New in Teams Calling, Meetings and Devices March 2024
What's New in Teams Calling, Meetings and Devices March 2024What's New in Teams Calling, Meetings and Devices March 2024
What's New in Teams Calling, Meetings and Devices March 2024
 
New from BookNet Canada for 2024: Loan Stars - Tech Forum 2024
New from BookNet Canada for 2024: Loan Stars - Tech Forum 2024New from BookNet Canada for 2024: Loan Stars - Tech Forum 2024
New from BookNet Canada for 2024: Loan Stars - Tech Forum 2024
 
Transcript: New from BookNet Canada for 2024: BNC CataList - Tech Forum 2024
Transcript: New from BookNet Canada for 2024: BNC CataList - Tech Forum 2024Transcript: New from BookNet Canada for 2024: BNC CataList - Tech Forum 2024
Transcript: New from BookNet Canada for 2024: BNC CataList - Tech Forum 2024
 
Transcript: New from BookNet Canada for 2024: Loan Stars - Tech Forum 2024
Transcript: New from BookNet Canada for 2024: Loan Stars - Tech Forum 2024Transcript: New from BookNet Canada for 2024: Loan Stars - Tech Forum 2024
Transcript: New from BookNet Canada for 2024: Loan Stars - Tech Forum 2024
 

Chapter 5 Thomas 9 Ed F 2008

  • 1. CrnprpR Applications of Integrals OYERVIEW Many things we want to know can be calculated with integrals: the areasbetweencurves,the volumes and sudaceareasof solids, the lengthsof curves, the amount of work it takes to pump liquids fiom belowground, the forces againsr floodgates,the coordinatesof the points where solid objects will balance.We define all of these as limits of Riemann sums of continuous functions on closed intervals. that is, as integrals,and evaluatethese limits with calculus. There is a pattern to how we define the integrals in applications, a pattern that, once learned,enablesus to define new integralswhen we need them. We look at specific applications 0rst, then examine the pattern and show how it leads to integrals in new situations. AreasBetweenCurves This section shows how to lind the areas of regions in the coordinate plane bv integrating the functions that define the regions, boundaries. The Basic Formula a Limitof Riemann as Sums Supposewe want to find the area of a region that is bounded above by the curve -y =.l(r), below by the curve ) = g(.r), and on the left and right by the lines .r : 4 and r : D (Fig. 5.1). The region might accidentallyhave a shapewhose area w e c o u l d f i n L w i r h g e o m e t r l b u r i l / a n d g a r e i t r b i l r a r ) o n r i n u o ut,u n c r r o n,r l . c e usuaily have to flnd the area with an integral. To see what the integml should be, we lirst approximate the region with,? basedon a parlition p : {.r0, ... , r,,} of [ri, D] (Fig. 5.2, on verticalrectangles rr, the following page). The area of the tth rectangle(Fig. 5.3, on rhe followrng page; ls AA1 : tsjcfi1 x width : Ll (c.t) g(ci)l A4. We then approximatethe area of the region by adding the areasof the ,?rectanqles: A* f ^A, -Itr,,.r grr.,rlA,:,. ' The region between y: f(x) and As ll P | -+ 0 the sums on rhe right approachthe limit y: g ( x ) a n d t h e l i n e sx : a a n d x = b . _ g(,r)l d,r because /j t/(r) 365
  • 2. Chapter5: Applicationsof Integrals 356 T - s(ck) f(t) Stctll ::.: We approximate regionwith the perpendicular the x-axis. to AAk = areaof kth rectangle,f(c*) - g(cr) = height, lxk = width rectangles this integral' /and g arecontinuous. takethearea theregionto be thevalueof of We That is, lim )-l/(c,)-gtcrtlAxr - fo rf ,r,- A- Stxt]dx nP o-- J Definition with /(r) > g(x) throughout b], ihenthe area g [4, Ifl ancl arecontinuous the curves : /(*) andy : g(t) ftom a to b is the y of the regionbetween integral [/ - 8 | liom a to D: o[ rb U(x)- g(x)ldx. (1) A= J To apply Eq. (1) we take the following steps. How to Find the Area Between Two Curves Graph the cunes and draw a tepresentative rectangle. Thts rcveals l. which curve is / (upper curve) and which is g (1owercurye). It also helps find the limits of integration if you do not aheady know them. ,s'I ,=sinr 2. Find the limits of integration. I--''- 3. Write a formula for f(x) - g(x). Simplify it if you can. 4. lntegrate [f(x) - gQ, fron a to b Tl1e number you get is the area. H Find the area between lv = sec2 and y : 3i1;5 from 0 to z/4' x EXAMPLE 1 Solution Step t; We sketch the cutves and a vertical rectangle(Fig. 5.4). The upper curve is the gmph of /(;r) = secz;r: the lower is the graph of g(;) : sipI' Sfep 2; The limits of integration are already given: a :0' b = r /4' ,'.]].The regionin Example'lwith a - SteP3: f (x) - 8(x) = sec2;r sin't rectangle. typicalapproximating
  • 3. 5.1 AreasBetweenCurves 367 Step 4: {r' *- sinx)r/x : ftan + cos -r x]i/a lo'o f'.+l,to+:+ -t L f, Curves That Intersect When a region is detemined by curves that intersect, the intersection points give the limits of integration. - x2 Find the area of the region enclosedby the parabola ! :2 EXAMPLE 2 : -y. and the line y Solution Sfep t; Sketch the curves and a vertical rectangle (Fig. 5.5). Identifying the upper (-{,,f(-r) -x. The ,{-coordinates and the lower curyes, we take /(x) :2- g(r): xz arLd pointsare the limits of integration. of the intersecdon Step 2; We find the limits of integation by solving | :2 - xz and y : -1 ri- ( 1 ,1 ) multaneously for .r.' -x [ q r l L t/r1 r ) . u r d . q ( f r 2-xz : x2-x-2:0 R.rir.. (x+l)(x -2):0 Fxrr.' (a s(r)) -1, x: x :2. sot. The region runs from x : - 1 to.r : 2. The limits of integrationare a : - l, b : 2. Step 3: 5.5 The regionin Example with a 2 typicalapproximatingrectangle. Step 4 t2 f -2 -t12 - : o: rf,.,>e(x)ldx I t z + x - x z t d x = l z r + z - ,I l' JJ J-t I r / 4 8 / | 1 (o*r-r,l (-'*l*r/ ?qq - -'2 At 3 2 l Technofogy The Intersectionof Two Graphs One of the difficult and some- times frustrating parts of integmtion applications is finding the limits of inte- gration. To do this you often have to find the zeroes of a function or the intenection points of two curues. g(r) using a graphing utility, you enter To solve the equation /(x): yr: f@) and y2: g@)
  • 4. Chapter Applications lntegrals of 5: 368 you andusethe grapherroutine to find the Pointsof intersection'Altematively' - g(T) : 0 with a root finder'Try bothprocedures lun solve*tJ /(:r) quation with and g(x):3-x' f (x):lnx hidden When points of intersectionarc not clearly revealedor you suspect further use of calculus behavior,additional work with the graphing utility or may be necessary, ISECT =.79205996845 curves : Inx and lz:3 - x' usinga built-infundion y1 a) The intersecting intersection to find the - : x O) Usinga built-inroot finder to find the zero of f(x) Inx 3 + with ChangingFormulas Boundaries points' we partition If the formula for a bounding curve changesat one or mole th,gionintosubregionsthatconespondtotheformulachangesandapplyEq. (1) to eachsubregion. Find the areaof the region in the fint quadrantthat is bounded EXAMPLE 3 - 2' : aboveby y : .r/i and below by the t-axis and the line y x solution is the graphof Step t.' The sketch(Fig. 5.6) showsthat the region'supperboundary <2to g(x) = fromg(;r):0for0 S r changes f(; : JV. Theloweiboundary -' Z tot Z : t : 4 (thereis agreement r : 2)' We partition the region at r : 2 at rectanglefor eachsubregion' into subregiJns ani f and tketh a representative A 5.6 When the formulafor a boundingcurve changes, area integralchanges match to the (Example3). 'fe limits of integrationfor regionA area : O andb :,2' The lefchand step 2: li#t for region B rs a = 2. To find the right-hand limit, we solve the equations
  • 5. 5.1 AreasBetween Curves 369 y : rG andy : x - 2 simultaneously r.' for EqratcI (.r)rnd ^- - z v^ .9(11 a:(x.-2)2:x.2-4x+4 SqLrafeboth x 2- 5 x + 4 : o Re$'r'ite. (.r-1)(r-4):0 FacL(r'. -_1 -_i Solve. Only the value.x=4 satisfies equation the The valuer:1is an Ji:x-2. extraneous introduced squaring. right-hand root by The limit is b:4. f (x)- S@): -0: S t e p3 . ' F o r 05 x = 2 : - e ( x ) : , 6E - ( x - 2q : ) Ji - x +2 F o r2 a x a 4 : f(x) Step 4.' We add the areaof subregions and B to find the total area: A f)-f4 = T o t a fr e a | J i d x I l t J x - x + 2 t d x a -o - - - - / J Jz areaof B arca of A 'l2 f) f1 -2 14 fil.+l1*'''-v+2,1, /2 - ?ef,,_o_ (1,,0,',, .' 8)_ _8 r r , r{zt'''-z++1 ?,t, : !. -, , 33 lntegrationwith Respect y to If a region's bounding curves are describedby functions of y, the approximating rectanglesare horizontal insteadof vertical and the basic formula has y in place of r. t 1 I L*,,, use tJteformula In Eq. (2),/always denotes right-hand the - stytldy. e: l.' vrrt (2) curve and I the left-hand curve, so fO) - e(y) is nonnegative.
  • 6. Chapter 5: Applicationsof lntegrals 370 Find the area of the region in Example 3 by integrating with EXAMPLE 4 rcspect to ). Solution (c()), ).) x:y+2 Step t; We sketchthe region and a typical horiz,ontalrectanglebasedon a panition of an interval ofy values(Fig. 5.7). The region's right-handboundary is the line x : y f 2, so l (y) : y + 2. The left-hand boundary is the curve ir : y2, so g(y) : y2. Step 2: The lower limit of integration is l = 0. We find the upper lirnit by solving I : y *2 and r: )2 simultaneously y; for 5.7 lt takestwo integrations find the l+t:l to areaof this regionif we integrate with 2=o ,t-) respectto x It takes only one if we with respect y (Example integrate to 4). ( ) + 1 ) ( ) 2 ): 0 Iurt,, 1'- _t ) S ,i l . The upperlimit of integmtion D : 2. (The valuey : -1 givesa point of inter- is sectlor belowthe r-axis.) Step3: =)+2 y2=2iy )? /())-s()) Step 4: v,rt- : e: 1,,'' srt,rat 1 2 + y - y ' ) l d y l,' f r,2 rt l' : +; 1l L2Y 3 ln 8 l0 - o.+ t - 4 3 3 This is the result of Example 3, fbund with less work. Integrals with Formulas from Geometry Combining andgeometry. The fastest way to find an areamay be to combinecalculus The Area of the Region in Example 3 Found the Fastesi EXAMPLE5 way Find the area of the region in Example 3. So/ution The area we want is the areabetweenthe curye I : Ji,O = x S 4, and 1:rf the x-axis, minus the area of a triangle with base 2 and height 2 (Fig. 5.8): 2 lr2rr2r / x r u = f or t a , .2 '.2 2 ,11 J .lo 5.8 The areaof the blue regionis the :3ttl-o -'z:+' area underthe parabola = 1& minus , y the areaof the triangle. JJ -
  • 7. Exercises 5.1 371 Moral of Examples3-5 It is sometimes easierto lind the areabetween two with respect y instead ,r. Also, it may help to combine curves integrating by to of geometry calculus. and After sketching region,takea moment determine the to the bestwav to oroceed. Exercises 5.1 Find the areasof the shadedregions in Exercises 1-8. l. 2.
  • 8. Chapter Applications Integrals of 5: 372 16. y : x 2 - 2 x and Y=r area. In Exercises findthetotalshaded 9-12, and y = -tt2 14x t7. ) : 12 10. 9, ] 18. y = l - l x ' ano ]:x-14 (-3,5) 19. y = x 4 - 4 x 2 + 4 and Y=a2 I and y:0 20, y = r n @ = 7 , a>0, _,_,2 5y = x * 6 (How many intersection points are 21. y = {fl nd there?) 2x3 - (x212)+4 5x 22,y:lx2-41'and ): 2 I Find the areasof the regions enclosedby the lines and curves 23-30. Exercises -3) : r : 0 , a n d) : 3 x:2y2, 23. 24. x = y2 u1t4 a: y 12 and' 4x - Y = 16 25. y2 - 4x:4 26, x - y2 :0 arld x +2Y2 =3 ^nd r+3Y2:2 2 7 ,x * y 2 : O and ,r*)a:2 2 8 .x - y 2 / t : 0 (-2,4 y 29.x =y2-l and x='yJl 3 0 . r = ) l 3- y 2 a n d x = 2 y -2 -l Find the areasof the regions enclosedby the curves in Exercises I 31'34. and .xa-Y:l 31.4x2+y:4 I a n d 3 x 2- y : 4 y :O 3 2 .x 3 I, (3,5) for xZ0 and :r*)a:1, 3 3 .x * 4 y 2 : 4 and 4.t*y2:0 3 4 ,x a y z : 3 12. Find the arcas of the regions enclosedby the lines and curves in ) I+. 35-42. Exercises (3,6) 6 and y = sin2.r, 0:.x57t 3 5 .y = 2 s 1 n x and y:5e92.r, -7t13=x Sr13 v 36.):8cos.r .t2 and 1:1 3 7 .y : c o s ( r x / 2 ) 38. y = sin(nx/2) and 1t:r (3,1) and'x:tt/4 39, l:sec2.r, y=tarfx, x:-tr/4, and .x:-tan2), -7r14=r 3n/4 4O. :ta#y x r and -x=0, 0=y<7t/2 4 1 .r : 3 s i n l / i o s y -' 4 2 .y : s s s 2 1 1 t a J 3 1 r r d y : r 1 / 1 , - l = i r : l 3) a rcgion enclosed the curve by 43. Find the areaof the propeller-shaped Find the areasof the regions enclosedby the lines and curves in - y3 : 0 andthe line.r - Y : 0. t 13-22. Exercises region enclosedby the 44. Find the area of the propeller-shaped 1 3 . y : a 2- 2 clruesx -yll3 : 0 andr - )l/5 = 0. ar'd y:2 14. y =2x - x2 arLd Y: -J 45, Find the areaof the region in the first quadrantboundedby the line I :1, the line,t :2, the curve1 : 1/-r'?, the -t-aris. and 15. y :1+ and ) :8r
  • 9. Exercises 5.1 373 Suppose area of the region betweenthe graph of a positive 46. Find the area of the tdangular region in the first quadrant the continuousfunction / and the .x-axisfrom x : 4 to x : b is 4 boundedon the left by the y-axis and on the right by the curves units. Find the areabetween curves)r: /(.r) and the ) = sin.rand): cos,r. square y-2f(x)ftomx:atox-b. belowby the pambola : ,r2 andabove by 47, The regionbounded ) Which of the following integals, if either calculates areaof the : 4 is to be parlitioned into two subsections equal of the line ) region shownhere?Give reasonsfor your answer. areaby cutting acrossit with the horizontal line ), = c. the shaded a) Sketchthe region and draw a line ) : c acrossit that looks fL f) -l-x)d:(= I (x 2xdx al | about right. In terms of c, what are the coordinates the of J J-t I points where the line and parabolaintersect?Add them to ' rot tl -zxax bt / r-x-(r))dx- your figure. | JI J-t b) Find c by integratingwith respectto ),. (This puts c in the limits of integration.) to.x.(Thisputsc into the c) Find c by integntingwith respect integrand well.) as ''2 and Find the areaof the regionbetween curve )):3 the the line y = 1 by integratingwith respectto (a) t, (b) ). 49. Find the areaof the region in the first quadrantboundedon the left by the ),-axis,below by the line y = t /4, aboveleft by the curve y = I * .,[, and aboveright by he (i.jtr..te : 2/ Ji, y 50. Find the area of the region in the first quadrantboundedon the left by the y-axis, below by the curve r : 2/t, aboveleft by right by the line i :3 - ). the curyer : () - l)'?,andabove 54. True, sometimestue, or never tue? The area of the region between graphsof the continuous functions): /(i) and the : g(r) andthe verticallinesr : a andx: b (a < b) rs ) rb I lf @) - s(x)ld)c. J Give reasonsfor your answer. S CASExplorationsand Proiects In Exercises 55-58, you will find the area between curves in the plane when you cannot find their points of intersectionusing simple algebra. Use a CAS to perform the following steps: The figure here shows triangle AOC inscribed in the region cut a) Plot the curves together to see what they look like and how many ftom the parabola ) - .r2 by the line y: a2. Find the limit of (hey hae. points of inrersection the latio of the area of the triangle to the area of the parabolic b) Use the numerical equation solver in your CAS to find all the region as 4 approaches zerc. points of intersection. - g(r)l over consecutive pairs of intersection c) Integate f(r) values. d) Sum together the integrals found in part (c). ^- ^ -2, I . g 1 x .:1 - J x 5 5 . '/3 2 3 : 1x1 t4 1 0 .g ( . r r : 8 - l 2 r 5 6 ./ t x t : i - t r ' - 57. /(r) =x *sin(2tc), g(x) : 13 58. /(.x) : -r2sqsir, g(tc) : x1 - x
  • 10. Chapter 5: Applicationsof Integrals 374 Volumes Slicing by Finding From the areasof regions with curved boundaries,we can calculatethe volumes of cylinders with curved basesby multiplying base area by height. From the volumes of such cylinders, we can calculate the volumes of other solids. 5licing Supposewe want to find the volume of a solid like the one shown in Fig. 5.9. At Cross sectionR(jr). Its area is A(ir). each point r in the closed inteNal la, rl the cross section of the solid is a region R(x) whose area is A(r). This makesA a real-valuedfunction of .r' If it is also a continuous lunction of .r, we can use it to define and calculate the volume of the solid as an integral in the following way. We partition the interval [4, ]l along the r-a{is in the usual manner and slice the solid, as we would a loaf of bread, by planes petpendicular to the -r-axis at the partition points. The tth slice, the one between the planes at xk r and tn, has approximatelythe same volume as the cylinder betweenthesetwo planes basedon the region R(,rr) (Fig. 5.10). The volume of this cylinder is Vr : base area x height sedion lf the areaA(x) of the cross : A(rr) x (distancebetween the planes at -rr r and xr) functionof x, we can R(x)is a continuous find the volumeof the solid bY : A(xr)Axr. A(x) from a to b. integrating The volume of the solid is therelbre approximatedby the cylinder volume sum - r,. r,r-. ?-,'^'^ This is a Riemann sum for the function A(r) on [c, Dl. We expectthe approxlmahons from these sums to improve as the norm of the partition of la, bl goes to zero, so we define their limiting integral to be the volume of the solid. Approxrmatlng Plane at.r[ I cllinderbased p l a n en r r , on]((rt] -'i.---------' -, The cylinder's base is the region R(,v*). NOTTO SCAI-E view of the sliceof the solidbetweenthe . Enlarged cylinder' planes xk r and xk and its approximating at
  • 11. 5 . 2 F i n d i n g o l u m eb y S l i c i n g 3 7 5 V s Definition The volume of a solid of known integrable cross-sectionarea A(x) from x = a to x: D is the integralofA from a to r. v : fo e61a,. (j) J T o a p p l l E q . { l ) . w e r a k et h e f o l l o w i n g: r e p . . How to FindVolumesby the Method of Slicing 1 Sketch the solid and a typical cross section. 2. Find a formula for A(r). 3. Find the limits of integration. 4. Integrate A(x) to find the volume. EXAMPLE 7 A pyramid 3 m high has a square base that is 3 mon a side. The cross section of the pyramid perpendicularto the altitude r m down from the ve ex is a squarer m on a side. Find the volume of the pyramid. Solution Step 1: A sketcll. We draw the pyramid with its altitude along the r-axis and its vertexat the origin and includea typical crosssection(Fig. 5.11). Typical cfoss section --r$€t -=_,,., sections the pyramidin Example The cross of 1 are 5quare5. Step 2: A.fonnula for A(x). The cross section at r is a squarex meters on a side, so lts area ls A(r) : 12' Step 3: Tlle lintits of integration. The squaresgo from r :0 to;r :3 Step 4: The volwne. 'l' lb t' - r. -, At^trlt - v=I | t:dr r |=9 J.. J,, .l , The volume is 9 mr.
  • 12. Chapter Applications Integrals of 376 5: wedge cut from a cylinderof radius3 by two planes. is 2 A curved EXAMPLE Bonaventura Cavalieri ('l 598-1647) planecrosses One planeis perpendicular the axis of the cylinder.The second to Cavalied, a studentof Galileo's, discovered the nrst plane at a 45' angle at the centerof the cylinder. Find the volume of the that if two plane rcgions can be afiangedto wedge. lie over the sameinterval of the i-axis 1n sucha way that they haveidentical vertical Solution crosssectionsat every point, then the rcgions perpen- Step 1: A sketch. draw the wedgeand sketcha typical crosssection We havethe sarnearea.The theorem(and a letter dicularto the:r-axis(Fig. 5.12). from Galileo) were of recommendation enoughto win Cavalied a chai at the Univelsityof Bolognain 1629.The solid 2lt -7 geometryversionin Example 3, which Cavalieri neverproved,was given his name by later geometen. have Crcsssections the samelongthat everypoint in [d, b] 5-t2 The wedgeof Example slicedperpendicular 2. sections redangles. are to the x-axis. The cross Step 2: Theformula for A(x). The cross section at r is a rectangleof area A(') : (heishtxwidth) (zJe - 'r) : (r) : z x t t: Y - x ' . ^, Step 3: The linxits of integration. The rectangles from r :0 to.r :3. run Step 4: The volune. ?b ^3 Ax)dx: | 2tcJ9x2dx - v:l Ja JO 113 : -;e - xr3/' )o I ,f : o+?e),/' 1! = lf r1r. nncsfalc- J . L n d s u b s t i l U r eb r c k . tr - 19 Cavalieri's Theorerrr EXAMPLE 3 Cavalieri's theoremsaysthat solids with equal altitudesand identical parallel cross- theorem. Thesesolids 5.13 Cavalieri's section areashave the samevolume (Fig. 5.13). We can seethis imrnediately from havethe samevolume.You can illustrate Eq. (1) becauset}Ie cross-sectionarea function A(.r) is the same in each case. f this yourself with stacks coins. of
  • 13. Exercises 5.2 377 Exercises 5.2 Areas Cross-Section In Exercises and 2, find a fomula for the arcaA(r) of the cross I sectionsof the solid perpendicular the l-axrs. to 1. The solid lies betweenplanesperpendicular the r-axis at i = to -l and,r:1. In eachcase,the crosssections perpendicular to the r-axis betweentheseplanesrun from the semicircley = -^4 - ,r2 to the semicircley : 4[ - ,2. a) The cross sectionsme circular disks with diametersin the xy-plane. planesperpendicular ther-axis at.r : 0 The solidliesbetween to andl : 4. The crosssectionsperpendicular the-t-axisbetween to these planes run from the parabola y = -aE to the parabola r: Ji. a) The cross sectionsare circular disks with diametersin the r)-plane. b) The crcsssections squares are with bases ther),-plane. in b) The crosssectionsare squareswith basesin the r)-plane. c) The cross sectionsarc squarcswith diagonalsin the xy- plane. (The length of a square'sdiagonalis .,4 times the lengthof its sides.) c) The crosssections squares are with diagonals ther)-plane. in d) The crosssectionsare equilateraltriangleswith basesin the rJ-plane. Volumes Slicing by Find the volumesof the solids in Exercises3-12. 3. The solid lies betweenplanesperpendicular the,-axis at .r : 0 to and r :4. The crosssections perpendicular the axis on the to The crosssectionsarc equilateraltriangleswith bases the in interval 0 5 i :4 are squareswhose diagonalsrun ftom the d) parabola1 : -.vE to the parabolaf : Jtr. ry-plane.
  • 14. 374 chapter5: Applications Integrals of 4, The solid lies between planes perpendicularto the r-axis at -x : - 1 and r : I . The cross sections perpendicular to the ir axis are run from lhe parabolay - x7 to circulardisks whosediamerers the Darabola :2 - t2. t 5. The solid lies betweenplanesperpendicular the r-axis at to Cavalieri's Theorem r : - I andr : 1. The crosssections perpendicular the axis to ll. A twisted solid. A square sidelengths lies in a planeper- of betweentheseplanesare vertical squares whosebase edges pendicular a line L Onevertex the square on t. As this to of lies run from the semicircle - - 1(1- rt lo rhe semicircle - y J squ&emovesa distance alongl, the square l? tums one revo- !7-7. lution aboutZ to generatea corkscrewlike column with square perpendicular ther-axis at r = 6. The solidlies between planes to crosssections. I and n: l. The crosssections perpendicular the ,r-axis to a) Find the volume of the column. planes squares between these are whosediagonals frcm the run b) What will the volumebe if the squaretums twice instead semicircle : -{ -7 to the semicircle y: JT=V. Qn ) Givercasons your answer of once? for lengthof a square's diagonal r/Z timesthelengthof its sides.) is berween curve) : 2 vfi; planes 12. A solid lies between 7. t he base thesolidis thereSion of lhe The qoss sections perpendicular tbe -{-axisat and the interval[0, r] on the.r-axis. perpen- to -r = 0 ard .r : 12.The cross dicularto the r-axis are sections planes perpendicular by to ther-axis arecirculardisks whosediametersrun from the line ) : -r/2 to the line ) : -r. Explain why the solid has the same volumeas a right circular conewith baseradius3 and height12. 13. Cavalieri'soriginal theorem. ProveCavalieri's original theo veftical equilateral triangles with bases running from the a) rem (marginalnote, page 376), assuming that each region is I-axrs to me culve: boundedabove belowby thegraphs continuous and of functions. b) vertical squareswith basesrunning from the r-axis to the t4. The volume of a hemisphere(a classical cutve. application af Cav- alieri's theorem). Derive the formula V : (2/3)zR3 for the 8. The solid lies between planes pe.pendicularto the r-axis at -x = volumeof a hemisphere radiusR by comparing crosssec- of its -r /3 and x : xr/3. The crcss sectionsperpendicularto the r- tionswith the crosssections a solid right circularcylinderof of axis are mdiusR andheightR from which a solidright circularconeof circular disks with diameters running from the curve y : a) baseradius andheishtR hasbeenremoved. R tan,r to the curve ): secr; verlical squares whose base edges run from the curve y : b) tanr to the cuNe ): SeCr. 9. The solid lies betweenplanesperpendicularto the y-axis at y : 0 and ) : 2. The cross sectionsperpendicularto the ]-axis are cir- cular disks with diametersrunning from they-axis to the parabola 10. The base of the solid is the disk j'? + ),2 5 1. The cross sections I and y - I by planesperpendicularto the )-axis between], - are isoscelesdght triangles with one leg in the disk.
  • 15. 5.3 Volumesof Solidsof Revolution-Disksand Washers 379 rn::rt sotids Revorution-Disks of E n The most common applicarion of the method of slicing is to solids of revolution. Solids of revolution are solids whose shapescan be generatedby revoJvingplane regions about axes. Thread spools are solids of revolution; so are hand weights and billiard balls. Solids of revolutign sometimeshave volumes we can find with formulas liom geometry, as in the case of a billiard ball. But when we want to find the volume of a blimp or to predict the weight of a part we are going to have turned on a lathe, formulas from geometry are of little help and we tum to calculus lor lhe un:* ers. 6 revolution Axisof revolution s # If we can arange for the region to be the region betweenthe graph of a contin- uous function I = R(r), a 5 x < ,, and the x-axis, and for the axis of revolution to be the x-axis (Fig. 5.14), we can find the solid's volume in the following way. The typical cross section of the solid perpendicularto the axis of revolution is a disk of radius R(x) and area A(.r) : 71136iut1: rlR(x)12. : The solid's volume, being the integral ofA from.r - a to x : b, is the integral of Cross sectionperpendjcularto zfR(x)1'zfrom a to D. the axis at r is a disk ofarea ,4(r) = 7r(radius)2 tt (R(,r))2 = Volume of a Solid of Revolution (Rotation About the .r-axis) The volume of the solid generatedby revolving about the.{-axis the region betweenthe -r-axis and the graph of the continuousfunction ) : R(r), a S :r 5 b, is v = [ , ftua;ur1t : [' r l R 1 x ) l 2 l r . a* (1) z J. J,, EXAMPLE I The region betweenthe curye y :.,rE,O <.{ S 4, and the r-axis and the x-axisfrom a to b about the x-axis. is revolved about the .x axis to generatea solid. Find its volume.
  • 16. 380 Chapter Applications lntegrals 5: of :l, Solution We draw ligures showing the region, a typical radius, and the generated solid (Fig. 5.15).The volumeis fb v : I trlR(x)'dx F - qr l r f1 : I rlrtTax /l1ft i Jo- (4t2 f4 - u . ,t 2 l a =tr lo x d x LJo =z-:8n. 1 l J ,, .f How to Find Volumes Using Eq. (1) 1, Draw the regionandidentifythe radiusfunctionR(x). 2. Square R(r) andmultiply by z. 3. Intesrate find the volume. to The axis of revolution the next example not the .x-axis, the rule for in is but calculating volumeis thesame: the Integmtelr(radius)2 betweenappropriate limits. (b) 5.15 The region(a) and solid(b) in EXAMPLE2 Find the volumeof the solid generated revolvingthe region by Exampl1.e bounded y : .rf andthe lines) : l, x:4 about line y : 1. by the Solution We drawfiguresshowing region,a typicalradius,andthe generated the solid (Fig. 5.16).The volumeis F f( lrr V: I r l 'R ( x ) 1 d r z J, r1 : I n l' J x _ t ) - d x Jt f4 : o I 'l , z E+ r ) a , Jt ^ .4 ^ tt tn lx- - + x l : -I :Tl 2':x ^ J o Lz lr -j To find the volume of a solid generated by revolving a region between the )-axis and a curve x : R()), c < l : d, about the y-axis, we use Eq. (i) with r replaced by ). Volume of a Solid of Revolution (Rotation About the y-axis) (b) 5.16 The region(a) and solid(b) in Example2. -
  • 17. 5.3 Volumesof Solidsof Revolution_Disksand Washers 391 EXAMPLE 3 Find the volume of the solid generatedby revolving Lnereglon betweenthe y-axis and the curve x:2/y,I S l,:4, aboutthe y-axis. Solution We draw figures showing the region, a typical radius, and the generated solid (Fig. 5.17).The volumeis v: :zln6)1'z ay l_ l (lr r l,o = ;) o, t4 /'r2 J, /11 I r r^ 4 T r'1. l3l I - tlt : qr | -: | : 4n r . t -t Jt v L )lr L4J .l EXAMPLE 4 Find the volume of the solid generatedby revolving the region betweenthe parabola :12 + I and the line I :3 aboutthe line; :3. I so/ut on We draw ligures showing the region, a typical radius, and the generated solid (Fig. 5.18). The volume is ( N1 ,: l.r r: ) J nrlR(y)'?dy pD /11r:1 rf -ll rl2 - y212 : dy I (b) /1 :r 5.17 The region(a) and solid(b) in L4-4y2+y4ldy I Example3. .,,51f : 7 r | 4 y - ; - y '+ a I 4 l J )l^ I 64rJ2 t5 R(])=3-()'?+ 1) ) z 0 -.'t r:t-+r I (a.) (b) 5.78 The region(a) and solid(b) in Example4. l
  • 18. Chapter 5: Applicationsof lntegrals 382 The WasherMethod If the region we revolve to generate a solid does not border on or cross the axis of revolution, the solid has a hole in it (Fig. 5.19). The cross sectionsperpendicular to the axis of revolution ate washersinstead of disks. The dimensionsof a typical washer are R(-r) Outerradius: , rx ) Inner radius: areais The washer's A(x) : TtlR(;)lz rVG)12: o (tn{)l' - tt(r)lt). - The Washer Fonnula for Finding Volumes fb .lrtx)f)dx v: I (3) ra ( [ R r x ) | ' : | | tl outer inner radius mdius squarcd squarcd ) = R(-r) Notice that the function integrated in Eq. (3) is t(Rz - r2), not T (R - r)'z. Also notice that Eq. (3) gives the disk method fomula if /(jr) is zero throughout [a, r]. Thus. the disk method is a soecial case of the washer method. The region boundedby the curve y : xz + | and the line y : EXAMPLE 5 -x t 3 is revolved about the -r-axis to generatea solid. Find the volume of the solid. 5.?9 The crosssectionsof the solid of revolution generated here are washers, Solution not disks, the integralf Ak) dx leads so Sfep t; Draw the region and sketcha line segmentacrossit perpendicularto the to a slightlydifferentformula. (the red segment Fig. 5.20). in a;dsof revolution Step 2,' Find the limits of integrationby finding the.r-coordinates the intersection of points. x2+l:-x+3 ,'+*-2:o (x+2)(x-l):0 , -_1 .-l Step 3; Find the outer and inner radii of the washerthat would be swept out by the line segmentif it were revolved about the x-axis along with the region. ftVe drewthe washer Fig. 5.21,but in your own work you neednot do that.)These in radii are the distances the endsof the line sesmentfrom the axis of revolution. of R(x) : -1 -1- 3 Outer radius: r(x):a241 Inner radius:
  • 19. 5.3 Volumes Solids Revolution-Disks Washers 383 of of and +-1 (1,2) lntegratlon j . . i T h er e g i o ni n E x a m p l 5 s p a n n e d y e b a line segmentperpendicular the axis to of revolution. when the regionis 5.21 The inner and outer radii of the cmss section Washer about the x-axis, line revolved the Outerradius: = r+3 washersweptout by the line segmentin R(r) will generatea washer. segment r(.r) = 12 + I Innerradius: Fi9.5.20. Step 4; Evaluate the volume integral. rb - ,= ([Rrx)l' lr(x)]') dx F ( t 1 - il J,, lt ^ = ({ x | 3)' tx' I lt'tdt I ltn!1 l /_rtr fl = / z(S 6x x2 xa;dx J-2 r' r'l' tllr -'l*t^ i sl, s LI How to FindVolumesby the WasherMethod Draw the region and sketch a line segmentacross it perpend.icularto l. the q,is of revolution. When the region is revolved, this segmentwill generatea typical washer cross section of the generatedsolid. 2. Find the limils of integration. Find the outer qnd inner rqdii of the washer swept out by the line segment. 4. Integrate to flnd the volume. To find the volume of a solid generatedby revolving a region about the )-axis, we use the stepslisted above but integratewith respectto l instead of ,r. The regionboundedby the parabola : x2 and the line y : 2y EXAMPLE 6 I quadrantis revolved about the )-axis to generatea solid. Find the volume in the first of the solid.
  • 20. 384 Chapter Applications Integrals 5: of Solution Step t.' Draw the region and sketch a line segment across it perpendicular to the axis of revolution, in this case the ),-axis (Fig. 5.22). Step 2: The line and parabola intersect at ) : 0 and l : 4, so the limits of inte- grationare c :0 andd :4. Step 3.' The radii of the washer swept out by the line segment are R(y) : a/ry, r(y) : y/2 (Figs.5.22 and 5.23). ? t22 The region,limitsof integration, 523 The washersweptout by the line and radii in Example 6. segmentin Fig.5.22. Step 4: Eq. (-l ) 'irh i - ,: , (tnci)l, tr(y)t2) dy 1 : l,^ -ltrJ')r, (ltl' sleps I .rnd -l :L'(,-';)*:l+-i]:, 8 3 EXAMPLE 7 The region in the first quadrantenclosedby the parabola y :2e2, the y-axis, and the line ) : I is revolved about the line,r : 3/2 to generatea solid. )=*2or*:f t 5ol Find the volume of the solid. 4!.=;-ti Solution Step 1.' Draw the region and sketch a line segment across it perpendicular to the Er axis of revolution, in this case the line ; :3/2 (Fig. 5.24). Step 2: Tbe limits of integration are ) : 0 to l : 1. Step 3.' The radii of the washer swept out by the line segment arc R(y) :3/2, 524 The region,limitsof integration, r(y) : (3/2) - -/1, (Fiss. 5.24 and 5.25). and radii in Example 7.